Combine Sample Papers

You might also like

Download as pdf or txt
Download as pdf or txt
You are on page 1of 114

CLASS 9 ENGLISH BLUEPRINT (2023-24)

LANGUAGE AND LITERATURE

*The students will be tested from the entire syllabus. (Term-1 + Term-2)

Section:A (Reading Skills) (20 Marks)

I Reading Comprehension through Unseen Passage


1. Discursive passage of 400-450 words. (10 marks)

2. Case-based factual passage (with visual input-statistical data/chart etc.) of 200-250


words. (10 marks)
(Total length of two passages to be 600-700 words) Multiple Choice Questions /
Objective Type Questions/Very Short Answer Questions will be asked to assess
comprehension, interpretation, analysis, inference, evaluation and vocabulary.

Section:B (Writing Skills and Grammar) (20 Marks)

II Grammar (10 Marks)


Determiners
Tenses
Modals
Subject – verb concord
Reported speech o
Commands and requests/Statements/Questions

3. The courses at the secondary level seek to cement high professional grasp of
grammatical items and levels of accuracy. Accurate use of spelling, punctuation and
grammar will be assessed through Gap Filling/ Editing/Transformation exercises. Ten
out of twelve questions will be attempted.

III Writing Skills (10 Marks)


4. Writing a Descriptive Paragraph (word limit 100-120 words), describing a person/
event/situation, based on visual or verbal cue/s.
One out of two questions to be answered. (5 marks)

5. Writing a Story (on a given cue/title)/Diary Entry, in 100-120 words. One out of two
questions is to be answered. (5 marks)
Section:C Language through Literature (40 Marks)

IV. Reference to the Context (5+5 = 10 Marks)

6. One extract out of two, from Drama/Prose.

7. One extract out of two, from poetry. Multiple Choice Questions/Objective Type
Questions will be asked to assess interpretation, analysis, inference, evaluation,
appreciation and vocabulary.

V. Short & Long Answer Questions


8. Four out of Five Short Answer Type Questions to be answered in 40-50 words from
the book BEEHIVE to assess interpretation, analysis, inference and evaluation.
(4x3=12 marks)

9. Two out of Three Short Answer Type Questions to be answered in 40-50 words from
the book MOMENTS to assess interpretation, analysis, inference and evaluation. (3x2=6
marks)

10. One out of two Long Answer Type Questions from BEEHIVE to be answered in about
100-120 words to assess creativity, imagination and extrapolation beyond the text and
across the text.
This can also be a passage-based question taken from a situation/plot from the text.
(6 marks)

11. One out of two Long Answer Type Questions from MOMENTS, on theme or plot
involving interpretation, extrapolation beyond the text and inference or character sketch
to be answered in about 100-120 words. (6 marks)
SAMPLE QUESTION PAPER (2023-24)
Class 9 ENGLISH (LANGUAGE AND LITERATURE)

Maximum Marks: 80 Time: 3 Hours

General Instructions:
● The Question Paper contains THREE sections: READING, GRAMMAR & WRITING
and LITERATURE.
● Attempt questions based on specific instructions for each part.

SECTION A: READING SKILLS (20 Marks)


Reading Comprehension through Unseen Passages

1. Read the following text: (10 Marks)


(1) Detachment is not physically renouncing our possessions, profession, home and
family as many of us think. It is the ability to let go of all the desires, habits, emotions,
thoughts, speech and behaviour patterns that are negative or detrimental to our well
being in any way. Detachment and discrimination are interdependent; they mutually
strengthen each other and empower us. The more we are able to discriminate, the more
we are able to segregate our harmful desires and discard them. The more mental trash
we discard this way, the more we empower our Faculty of Discrimination.

(2) There is a simple technique we can all practice to strengthen our discrimination and
detachment and get rid of unnecessary desires. This will generate the awareness we
need to control our senses and mind and make them stop churning out desires every
moment. To practise this technique, we should select a quiet place when we have some
leisure time.

(3) Sit down with a few sheets of paper and a pen or a computer and make two lists. In
the first list, understand and write down your most important goal or agenda in this life.
Then meticulously add all the other goals that you would like to achieve. While you make
this list, you should think only about yourself. Include what you would like to contribute
to the welfare of your dependents, children and their forthcoming generations.

(4) Once you complete this, set it aside and begin the second list. List all your desires
here.They may be big or small, important or insignificant, right or wrong,demonic or
divine. Just remove the lid on the Pandora's Box and allow them all to spill out freely.
Don't try to censor or edit the list of desires at this stage. Complete the list first. After you
have made an exhaustive list, go over it to see which of these desires are truly
necessary, conducive to your meditation, helpful and favourable to you and beneficial to
others.

(5) If you have been serious and honest in doing this exercise, you will have long lists of
goals and desires and many of them maybe selfish or detrimental to your goal of
meditation, visualisation, self realisation, peace, happiness and bliss.This exercise will
help you develop frankness and honesty with yourself. You will touch your true feelings.
Answer the following questions, based on the passage above.
a) Which of the following statements best describes the author’s attitude towards
detachment? 1
i. Detachment helps us physically renounce a position.
ii. Detachment strengthens our sense of discrimination.
iii. Detachment helps us get detached from family and relations.
iv. Detachment makes us let go of our desires.

b) What is the writer’s suggestion for the given line from paragraph-2? Rationalise your
response in about 40 words. 2
There is a simple technique we can all practice to strengthen our discrimination and
detachment.

c) Complete the sentence appropriately choosing appropriate words from the passage
to complete the author's advice.
The author encourages the readers to ‘remove the lid on Pandora's Box and allow your
desires and wishes to spill out freely’ __________________. 1

d) The passage includes some word pairs that are similar to each other from the sets
given below. Identify the word pair that is the odd one out. 1
i.Detachment and renunciation
ii. Detachment and discrimination
iii. Censor and edit
iv. Goal and agenda

e) Complete the sentence appropriately: 1


We can say that the author's tone becomes more neutral and objective when he
correlates discrimination and detachment because ______________________.

f) Based on your reading of the text, list two reasons why the writer implies that the
human mind is full of trash. Answer in about 30-40 words. 2
g) What is the message conveyed by Rohit’s experience in the following case? 1
Rohit wanted to be an IAS so, he stayed in a separate accommodation, away from his
family, to prepare well for his exam. He missed his family and felt guilty. When he cleared
the exam, he realised ______________________.
i. Detachment is discrimination.
ii. One should not be selfish for one’s dreams.
iii. Detachment is not complete separation.
iv. Emotions are more important than goals.

h) Supply one point to justify the following: 1


Awareness puts a lid on our mind which keeps turning out desires every moment which
can be confusing.

2. Read the following text. (10 Marks)


(1) The most characteristic feature of spiders is their ability to produce Silken threads.
Spiders normally have six finger - like Silk glands, called ‘spinnerets’ located beneath
their abdomen. To spin a web, the spider quizzes the silk out of its body through two
small holes. The liquid Silk is thicker than water and dries into a fine thread as soon as
it comes in contact with air. Spider silk is much stronger than steel and resists breakage
because of its elasticity. It can stretch up to 140% of its original length. The diameter of
these Silken threads is about 10,000 of an inch. Spiders use this silk to make webs to
trap their prey. The egg sacs of some spiders are also made of silk which pretreats the
unborn progeny. Sometimes the spider can also use its silk as a safety feature.A certain
species of spider ‘Phidippus Apacheanecs’ trails a safety line of fine silk as it jumps. It
uses the silk line to recover if it misses its target. The Silk of ‘Naphils Spider’ is the
strongest natural fibre known.

(2) The spinning of webs is a remarkable example of instinctive behaviour. A spider does
not have to learn how to make a web, although spinning itself can be adapted to unique
circumstances. For example, ‘Orb’ webs are circular in shape and take an hour to build.
Funnel spiders pin a net - like entrance to their homes, which helps to trap food. Similarly,
trapdoor spiders dig their homes and spin a trap door out of their silk for trapping insects.

Black and Yellow Garden Spider Fisher Spider


Do not need webs to catch prey, track
Typically found in fields and gardens down insects and Pounds on them.
during daytime.
Common House Spider Woodlouses Hunting Spider

Spotted outside in leaves wood or Rock


Found in houses shirts and garridge pill bugs.
typically weave their webs in ceiling and
window.

Grass Spiders

Webs are weaved in grass, visible in


morning dew.
(3) As integral components of our biosphere, spiders have an intrinsic existence value.
They are efficient predators that play an important role in controlling the pests of cotton.
They are also a source of food for other creatures such as birds and wasps.Thus, the
presence of spiders is an ecological indicator signalling the health of the natural
ecosystem.

Answer the following questions, based on the passage.


a) Complete the following analogy appropriately, based on your understanding of
paragraph 2. 1
Funnel spiders and trapdoor spiders build structures to trap insects for food, is
comparable with a rat-trap set for a mouse, but with a big difference
__________________________________________.

b) Fill in the blanks with the appropriate option from those given in brackets, based on
your understanding of paragraph 1. 1
Spider Silk is much stronger than steel and resists breakage because of its elasticity. It
can stretch up to 140% of its original length is ___________ a fact/an opinion because
it is a ____________ (subjective judgement/objective detail).

c) Justify the following in about 30-40 words. 2


Spiders are great indicators of a healthy ecosystem.

d) Based on the passage, name two species each of spiders found in our homes and
outdoors. 1

e) What kind of behaviour does the author say the spiders exhibit when at work? State
any one inference that can be drawn from this. 1

f) Select the option that correctly displays what ‘threads’ signify as per the passage. 1
i.
ii.

iii.

iv.

g) Infer one benefit and one drawback of spiders whether at home or in the garden.
Answer in about 30-40 words. 2

h) Which of the following is an incorrect fact about spiders from the study mentioned in
the passage? 1
A. The egg sacs of some spiders are also made of silk
B. Woodhouse Hunting spiders live on leaves and wood.
C. Spinning can be adapted to unique circumstances.
D. Naphils spider weaves the silliest web known.

SECTION B: WRITING SKILLS & GRAMMAR (20 Marks)

GRAMMAR (10 Marks)


3. Complete ANY TEN of twelve of the following tasks, as directed.
i. Fill in the blanks by using the correct form of the word in the bracket, for the given
portion of a letter. 1
Dear Sir
Subject: Potholes in our locality are causing difficulties.
With due respect, I would like to inform you that we __________ (face) a lot of difficulties
due to potholes in roads in our area. Monsoon season arrived a few days ago and is
increasing our problems.

ii. Read the given sentence from an article on air pollution in Delhi. Identify the error and
supply the correction in the sentence. 1
Air quality in Delhi improves, but still remains in a ‘very poor’ category as smog
enveloped the city.

Use the given format for your response.


Error Correction

iii. John said to Jacob, “Are you feeling well today?” 1


Report the question asked by John.

iv. Select the correct option to complete the narration of the dialogue between Sonu and
his mother. 1
Sonu: Mother, I am going out with my friends for a while.
Mother: Be home by 8:00 p.m. for dinner.
When Sonu informed his mother that he ______________ for a while, his mother asked
him to be home by 8:00 p.m. for dinner.
A. will be going out with his friends
B. was going out with his friends
C. would be going out with his friends
D. he and his friends are going out

v. Fill in the blank by choosing the correct option, to complete the opening line of an
article on the rising awareness about millets internationally. 1
The United Nations declared 2023 as the International Year of Millets - to pay tribute to
___________ health and nutritional benefits.
A. its
B. it's
C. there
D. their

vi. Identify the error in the given newspaper headline and supply the correction.
Athletics Star helps State Stays Ahead of Competition. 1

Use the given format for your response.


Error Correction

vii. Select the option that identifies the error in a museum plaque and supplies the
correction for the same. 1

These building, inaugurated on 29 March 1954, was a former residence of a Maharaja.

Option No. Error Correction


A These This
B building buildings
C on at
D of by

viii. Complete the given narrative by filling in the blank with the correct option.
Let’s meet another day since neither Sam nor Savi _________ free today as they have
badminton practice. 1
A. were
B. are
C. be
D. is

ix. Read the conversation between two friends. Complete the sentence by reporting the
conversation. 1
He said, “I cannot help you at present because I am myself in difficulty”.
Freddy, “You are my last hope”.
Hemant told Freddy that he could not help him at present because he himself was in
difficulty, to which Freddy replied that he ____________ hope.
x. Complete Nelson Mandela’s quote by filling the blank with the correct option. 1
“The greatest glory in living ………. not in never falling, but in rising everytime we fall.”
A. lie B. lies C. lays D. laying

xi. Select the correct option to fill in the blanks for the given line, from a health magazine.
1
It must also be conceded that with effective packaging, we are able to enjoy food items
which __________ impossible otherwise.
A. were B. had been C. would have been D. was

xii. Identify the error and supply correction for the following from a news report. 1
Over the recent years many skilled craftsmen and women have lost their jobs inside the
pottery trade in the UK, but gradually English porcelain is regaining its reputation.

Use the given format for your response.


Error Correction

WRITING (10 Marks)


Note: All details presented in the questions are imaginary and created for assessment
purposes.

4.(A) You had an opportunity to meet Ruskin Bond recently, when your school organised
“Book Week”. Write a short description of him in not more than 100 to 120 words.

Hints: Born in Kasauli,India 19 May,1934 - Indian author of British descent- icon among
Indian writers - children's author - a top novelist - first novel, “The Room on the Roof”,
when he was seventeen - won him John Llewellyn Rhys Memorial prize in 1957 -
1992:Sahitya Akademi Award for English writing, for his short stories collection, “Our
Trees Still Grow in Dehra” - awarded the Padma Shri in 1999 for contributions to
children's literature - now lives with his adopted family - Landour near Mussoorie.

OR 5

(B) Last week you went to an Adventure Park “Rangmanch” in Gurugram as a part of a
school picnic with your friends and teachers. You felt exhilarated after the journey. Write
a page in your diary describing your experience. (100-120 words)

5. (A) Write a creative short story in 100-120 words, with a suitable title using the given
hints:
One day I was playing cricket - an old man fainted - had learnt first aid - called
ambulance - saved his life - every life is precious - presence of mind - courage

OR 5

(B) Complete the given story in 100-120 words, by using your own imagination and
creativity.
It is a freezing cold night. The doorbell rings at “Oberoi Villa”. Mr Oberoi opens the door
and finds a homeless family at his doorstep. He invites them into his home to sleep. But
in the morning the family doesn't leave.........

SECTION C: LITERATURE TEXTBOOK AND SUPPLEMENTARY READING TEXT


(40 Marks)

6. Read the given extracts and answer the questions for ANY ONE of the two,
given.
(A) My mother then ran out of the house carrying a small metal box full of important
documents. She dropped the case on the lawn and, in a crazed state, ran back into the
house. I knew what she was after. My father had died when I was young, and I was
certain that she is not going to let his pictures and letters go up in flames.They were the
only things that she had to remember him by. Still I screamed at her, “Mom! No!”
-(A House is not a Home)

i. Comment on the narrator‘s state of mind on seeing his house on fire? (30-40 words)1
ii. Presence of many fire trucks at the sight shows: 1
A. the efficiency of the department
B. the easy accessibility to the firemen
C.the gravity of the situation
D. All of the above
iii. Give another word similar to the meaning of ‘crazed’, used in the above extract. 1
iv. What character traits of the author are reflected when he screams at his mother? 2

OR

(B) GERRARD: That's a disguise outfit; false moustaches and what not. Now do you
believe me?
INTRUDER: (musingly) I don't know.
GERRARD: For God's sake clear that muddled head of yours and let's go. Come with
me in the car. I can use you. If you find it's a frame, you've got me in the car, and you've
still got your gun.
INTRUDER: Maybe you're right.
GERRARD: Then don't waste time. (goes and picks up hat and bag)
INTRUDER: Careful, Boss I am watching you.
-(If I were You)
i. Gerrard was looking for a disguise outfit ……….. 1
A. to save himself from being caught
B. to save himself from boredom
C. to save himself from spending money
D. to save himself from being killed.
ii. The phrase “muddled head”, used by Gerrard refers to someone who is
......................... 1
iii. With what qualities did Gerrard manage to convince the Intruder of his plan. Explain
in 30- 40 words. 2
iv. The criminal has been called an intruder all through the play. The meaning of intruder
is ........................ 1

7. Read the given extracts and answer the questions for ANY ONE of the two,
given.

(A) No motion has she now, no force - She neither hears nor sees,
Rolled round in earth's diurnal course with rocks and stones and trees.
i. By the use of the words like “Earth's diurnal course" the poet means................... 1
ii. ‘She’ in the above lines refers to ………… 1
A. poet’s daughter
B. poet’s wife
C. poet’s beloved
D. poet’s pet
iii. Comment on the line “No motion has she now, no force, she neither hears nor sees”.
(30-40 words) 1
iv. What is the tone of the poet reflected by the given lines? 2

(B) Every tinkle on the shingles Has an echo in the heart;


And a thousand dreamy fancies Into busy being start,
And a thousand recollections Weave their air-threads into woof, As I listen to the patter
Of the rain upon the roof.

i. What makes an echo in the poet's heart? 1


ii. Explain in your own words how the tinkle of the raindrops impacted the poet's heart.
(30-40 words) 2
iii. The sound of the raindrops on the roof is..........to the poet? 1
A. worrying B. gratifying C. despairing D. mind numbing
iv. Give an example of onomatopoeia from given lines. 1

8. Answer ANY FOUR of the following five questions, in about 40-50 words.
(4x3= 12)
i. According to the poet, how can a tree be killed completely?
ii. Albert Einstein was a visionary. Comment with reference to “A Truly Beautiful Mind.”
iii. Describe the atmosphere at the Baudhnath Stupa.
iv. Give any two reasons for the poet's wish to go to Innisfree.
v. Why does the doctor say he made another earth shaking decision? Why does he call
it “earth shaking’’?

9. Answer ANY TWO of the following three questions, in about 40-50 words.
(2x3= 6)
i. Mahendra was lucky to have Iswaran as his cook. Explain to support your answer.
ii. How was Behrman able to create a masterpiece?
iii. The Swallow knew he would not be able to survive the bitter cold of the city, but still
stayed back. Why?

10. Answer ANY ONE of the following two questions, in about 100-120 words.
(A) Kalam's boyhood friends were all Hindus. They were very close to each other. Based
on your reading of the chapter ‘My Childhood’, compose a speech to be delivered in the
morning assembly on the topic: “Importance of Communal Harmony”.

OR 6

(B) Maria Sharapova and Santosh Yadav faced many challenges in their journey to
stardom. Compare their characters and highlight the qualities that enabled both to
overcome difficulties. Add instances in support of your answer.

11. Answer ANY ONE of the following two questions, in about 100-120 words.
(A) You are the disciple in the story ‘In the Kingdom of Fools’. You are in trouble as you
are about to be hanged. You decide to write a letter to your Guru regretting your decision
to stay in the kingdom of fools and request him to help you. Write the letter in 100 to120
words.

OR 6

(B) Compare and contrast the character of Lushkoff the beggar and Lushkoff the notary
with the help of suitable instances from the chapter.
नमूना प्रश्न-पत्र – (2023-2024)
कक्षा-नवीं
ववषय-व दिं ी’ ब’

निर्धारित समय- 3 घंटे पर्


ू धांक-80

सामान्य निर्दे श-

इस प्रश्िपत्र में दो खंड हैं – खंड ‘अ’ औि ‘ब’।


1. खंड ‘अ’ में वस्तुपिक प्रश्ि पछ
ू े गए हैं।
2. खंड ‘ब’ में वर्ािधत्मक प्रश्ि पछ
ू े गए हैं।
3. निदे शों को बहुत सधवर्धिी से पढ़िए औि उिकध पधलि कीजिए।
4. यथधसंभव दोिों खंडों के प्रश्िों के उत्ति क्रमशः ललखखए ।

खंड-अ (वस्तुपरक प्रश्ि)

1.निम्िलिखखत गदयांश को पढ़कर पूछे गए प्रश्िों के सही उत्तर ववकल्पों से


चुनिए — (1x5=5)
कुछ लोग हि बधत पि ठहधके लगधकि हँसते हैं। उिसे पूछध िधए कक उिके हँसिे की विह
क्यध है , तो वे शधयद कोई खधस कधिर् िह ं बतध पधएँगे । कोई व्यजक्त ककसी एक घटिध पि
ककतिध कम यध ज़्यधदध हँसेगध यध केवल मुसकिधकि िह िधएगध यध होठों पि छलकती हँसी
को दबधकि िखेगध, यह सब उसके अपिे िीवि के अिुभवों से निर्धारित होतध है। बचपि से
ह घि कध मधहौल कैसध िहध है , घि में ककतिध हँसी-मजधक, बडों में क्यध होतध आयध है,
ककसके सधमिे हँसिध है यध िह ं हँसिध है— यह सब बच्चे शुरू से दे खते औि सीखते आते हैं,
पि ये ककसी सदस्य को यध अन्य को अपिध िोल मॉडल बिधकि उसके मत
ु धबबक़ अपिे
व्यजक्तत्व को बिधिध चधहते हैं।
पिे शधनियों के बधविद
ू िोजधिध थोडध हँसिे-हँसधिे कध अभ्यधस किते िहिे से आत्मववश्वधस व
सिु क्षध की भधविध ब़िती है। इससे ि केवल खद
ु को खश
ु ी महसस
ू होती है, बजकक आपस में
िड
ु े सब लोगों में खश
ु ी फैलती है। हँसिध हमधिे शधि रिक व मधिलसक स्वधस््य के ललए बहुत
लधभकधि है। इससे हमधिध इम्यि ू लसस्टम मजबत ू िहतध है औि बहुत-सी बीमधरियों से बचधव
हो सकतध है, लेककि जरूित से ज़्यधदध यध अिचु चत ति क़े से हँसिध दस
ू िों को पिे शधि कि
सकतध है। हँसी-मजधक से टें शि रिल ज होती है, मि हलकध होतध है। हँसते िहिे से िीवि
की कडी वधस्तववकतधओं, लगधतधि थकधवट कध एहसधस, अज्ञधत डि, तिह-तिह की चचंतधओं
औि झँझ
ु लधहट से िधहत लमलती है।मि की खुशी व शधंनत से कधयाशजक्त व सि
ृ िधत्मकतध
ब़िती है। क्यों ि हम आपसी भेदभधव लमटधकि, अपिे संबंर्ों को मर्िु बिधिे की कोलशश
किें , तधकक एक-दस
ू िे से अपिे दख
ु -ददा , खश
ु ी, अपिी सफलतधओं के सख
ु द अिभ
ु व बधँट सकें
औि आपसी मेल-िोल व आत्मववश्वधस के सधथ हँसी-खश
ु ी से िी पधएँ।
(I)ककसी एक घटिा पर हँसिा या मस
ु कराकर रह जािा आदर्द ककस पर निर्भर करता है?
(क) घि के मधहौल पि
(ख) िीवि के अिभ
ु वों पि
(ग) ठहधके लगधकि हँसिे कध कधिर् ि बतध पधिे पि
(घ) ‘ककसके सधमिे हँसिध है’ उस पि
(ii) बच्चे अपिा व्यक्ततत्व ककसका अिक
ु रण करते हुए निलमभत करिा चाहते हैं?
(क) पधरिवधरिक सदस्य यध ककसी अन्य को अपिध आदशा बिधकि
(ख) परिवधि में िो हो िहध उसे दे खकि
(ग) परिवधि के बडे िो कि िहे हैं उसे सीखकि
(घ) उपयुाक्त में से कोई िह ं
(iii) परे शानियों में र्ी हँसिे-हँसािे का अभ्यास करिे पर ककस प्रकार िार्ाक्न्वत हो सकते
हैं?
(क) स्वयं खुशी महसूस कििे पि संबंचर्त सभी लोगों में खुशी फैलती है।
(ख) आत्मववश्वधस ब़ितध है।
(ग) सुिक्षध की भधविध में वद्
ृ चर् होती है।
(घ) उपयुाक्त सभी
(iv) हँसते रहिे से जीवि की ककि वास्तववकताओं से राहत लमिती है ?
(क) टें शि रिल ज होिे पि मि हलकध होिे से
(ख) थकधवट, अज्ञधत डि,चचंतधओं ,झँझ
ु लधहट से
(ग) अिुचचत तिह से हँसिे से
(घ) मि की खुशी में शधंनत से
(v)निम्िलिखखत कथि (A) तथा कारण (R) को ध्यािपूवक
भ पदढ़ए। तत्पश्चात दर्दए गए
ववकल्पों में से कोई एक उचचत ववकल्प चि
ु कर लिखखए ।
कथि (A): भेदभधव भल
ु धकि संबर्
ं ों को मर्िु िह ं बिधयध िध सकतध।
कारण (R): मि की खश
ु ी व शधंनत से कधयाशजक्त औि सि
ृ िधत्मकतध ब़िती है।
(क) कथि (A) तथध कधिर् (R) दोिों ग़लत हैं।
(ख) कथि (A) ग़लत हैं ,लेककि कधिर् (R) सह है।
(ग) कथि (A) सह है, लेककि, कधिर् (R) ग़लत व्यधख्यध कितध है।
(घ) कथि (A) तथध कधिर् (R) दोिों सह है तथध कधिर् (R) कथि (A) की सह व्यधख्यध
कितध है ।
2.निम्िलिखखत गदयांश को पढ़कर पछ
ू े गए प्रश्िों के सही उत्तर ववकल्पों से
चनु िए — (1x5=5)

भधितीय संववर्धि सभध द्वधिध हमधिे नतिं गे कध िं ग, आकधि, बिधवट, िं गों कध क्रम आढद
निर्धारित ककयध गयध है । 22 िल
ु धई 1947 को संववर्धि सभध िे इसे स्वीकृत ककयध तथध 15
अगस्त 1947 को 31 बंदक
ू ों की सलधमी के सधथ पहल बधि ध्विधिोहर् ककयध गयध। तब से
आि तक इसे दे श की िधष्ट्र य स्वतंत्रतध, िधििीनतक संप्रभुतध, सधंस्कृनतक एकधत्मकतध एवं
सधमधजिक एकिसतध के प्रतीक रूप में फ़हिधयध िधतध है। हमधिध नतिं गध सौंदया, गौिव एवं
संवेदिशीलतध के प्रतीक को अपिे में समेटे है। इसकध केसरियध िं ग नि:स्पह
ृ त्यधग, बललदधि
कध प्रतीक है। मध्य में जस्थत श्वेतपट्ट सत्य के प्रकधश की प्रतीक है औि हिध िं ग प्रकृनत से
हमधिे िुडधव व र्िती से िुडी हमधि समद्
ृ चर् कध सच
ू क है। नतिं गे के मध्य अशोक चक्र है,
िो न्यधय की र्धिधवधढहकतध व गनत कध प्रतीक है। इसे र्मा चक्र भी कहते हैं। हमधिध नतिं गध
भी कहतध है कक इसकी छधयध में िह िहे लोगों कध पिू ध ववश्वधस है कक सत्य एवं र्मा(सद्गुर्)
ह मिुष्ट्यतध के नियधमक लसद्र्धंत हैं।
नतिं गध हमेशध हथकते, हथबि
ु े, सूती, िे शमी यध ऊिी कपडे (खधद ) कध होिध चधढहए। यह
आयतधकधि होतध है। इसकध अिुपधत 3 : 2 होिध चधढहए। अशोक चक्र िीले िं ग कध होतध है;
जिसमें समधंति 24 शलधकधएँ होतीं हैं । ध्विधिोहर् कििे वधले व्यजक्त को ध्वि से तीि
कदम दिू खडे होिध चधढहए। हि दशाक को तिकि, दोिों हधथों की मट्
ु ठी बधँर्े खडध होिध
चधढहए। इसी समय िधष्ट्रगधि बिधयध िधतध है , अतः हि श्रोतध नतिं गे के सम्मधि में अपिी
िगह जस्थि खडे हो िधिध चधढहए। यह ध्विधिोहर् िधष्ट्र यतध कध सम्मधि है । इस समय
हमधिध मिोभधव पववत्र औि आचिर् संयत होिध चधढहए।
आप िब भी नतिं गे को फ़हिधतध दे खें तो अपिे आप से प्रश्ि जरूि किें —क्यध एक स्वतंत्र,
स्वधलभमधिी िधष्ट्र के सम्मधनित िधगरिक होिे के िधते हम ति, मि, वचि औि कमा से
उसके प्रनत समवपात हैं?
(I) 22 जुिाई 1947 को संववधाि सर्ा िे तया स्वीकृत ककया?
(क) 15 अगस्त 1947 को ध्विधिोहर्
(ख 15 अगस्त 1947 को 31 बंदक
ू ों की सलधमी
(ग) नतिं गे कध िं ग, आकधि, बिधवट िं गों कध क्रम आढद के निर्धािर् को
(घ) (क) औि (ख)
(ii) नतरं गे की छत्रछाया में रह रहे िोगों की आस्था है।
(क) अशोक चक्र ह र्मा चक्र है।
(ख) नतिं गध सौंदया, गौिव व संवद
े िशीलतध कध प्रतीक है।
(ग) हिध िं ग समद्
ृ चर् सूचक है।
(घ) सत्य एवं र्मा ह मिष्ट्ु यतध के नियधमक लसद्र्धंत हैं।
(iii) नतरं गे की ववशेषता िहीं है ।
(क) यह आयतधकधि होतध है।
(ख) इसकध अिप
ु धत 2 : 3 होिध चधढहए।
(ग) यह हथकते, हथबि
ु े, सत
ू ी, िे शमी यध ऊिी कपडे कध होिध चधढहए।
(घ) अशोक चक्र में समधंति 24 शलधकधएँ होिी चधढहए।
(iv) ‘ध्वजारोहण’ प्रनतष्ठा है।
(क) संप्रभुतध की
(ख) सधमधजिक एकिसतध की
(ग) सधंस्कृनतक एकधत्मकतध की
(घ) िधष्ट्र यतध की
(v) निम्िलिखखत कथि (A) तथा कारण (R) को ध्यािपूवभक पदढ़ए। तत्पश्चात दर्दए गए
ववकल्पों में से कोई एक उचचत ववकल्प चि
ु कर लिखखए ।
कथि (A): 15 अगस्त 1947 को पहल बधि ध्विधिोहर् ककयध गयध थध।
कारण (R): हमें इस ढदि स्वतंत्रतध लमल थी।
(क) कथि (A) तथध कधिर् (R) दोिों ग़लत हैं।
(ख) कथि (A) ग़लत हैं ,लेककि कधिर् (R) सह है।
(ग) कथि (A) सह है, लेककि, कधिर् (R) ग़लत व्यधख्यध कितध है।
(घ) कथि (A) तथध कधिर् (R) दोिों सह है तथध कधिर् (R) कथि (A) की सह व्यधख्यध
कितध है ।
व्यावहाररक व्याकरण
3.दर्दए गए ववकल्पों में से उचचत ववकल्प चुिकर उत्तर
र्दीक्जए:- (1×2=2)

(i) निम्िलिखखत में से शब्र्द के लिए सही उर्दाहरण छाँदटए—


(क) वपतधिी अखबधि प़िते हैं।
(ख) वपतधिी बेटे को डधँट लगध िहे हैं।
(ग) बेटध
(घ) वपतधिी डधँटते हैं।

(ii) ‘मैं पत्र लिखूँगा।’ वातय में ‘पत्र’ तया है ?


(क) शब्द
(ख) पद
(ग) वर्ा
(घ) अक्षि

4. निम्िलिखखत में से ककन्हीं र्दो प्रश्िों के उत्तर


र्दीक्जए— (1×2=2)

(i).निम्िलिखखत में से ककस शब्र्द में अिस्


ु वार का प्रयोग हुआ है ?
(क) हं स (ख) हँस (ग) प्रधतः (घ) चधँदिी

(ii). निम्ि में से ककस शब्र्द में अिुिालसक का प्रयोग होिा चादहए?
(क) कधच (ख) नतिगध (ग) पकि (घ) मत्र

(iii).निम्िलिखखत में अिस्


ु वार का प्रयोग होगा?
(क) र्ुआ (ख) कचि (ग) ऊट (घ) आर्ी

5.निम्िलिखखत में से ककन्हीं चार प्रश्िों के उत्तर


र्दीक्जए— (1×4=4)

(i). ‘अचधकोष’ शब्र्द में ककस उपसगभ का प्रयोग हुआ है ?


क) अचर् (ख) अर् (ग) अ (घ) अचर्क

(ii). ‘र्दरु ’् उपसगभ से बिे शब्र्द को चनु िए—


(क) दब
ु ाल (ख) दरू ियधँ (ग) दरु
ु स्त (घ) दिू दशाि

(iii). ‘महािता’ शब्र्द में प्रयुतत प्रत्यय छाँदटए—


(क) महधि (ख) िेतध (ग) तध (घ) अतध

(iv). निम्िलिखखत में से कौि-सा शब्र्द ‘र्दार’ प्रत्यय से िहीं बिा है —


(क) समझदधि (ख) शधिदधि (ग) दक
ु धिदधि (घ) उदधि

(v). ‘पहाडी’ शब्र्द में प्रत्यय है —


(क) प (ख) हडी (ग) ई (घ) अि
6. निम्िलिखखत प्रश्िों में से ककन्हीं तीि प्रश्िों के उत्तर
र्दीक्जए— (1×3=3)

(i). ‘र्ारतेंर्द’ु का संचध ववग्रह है —


(क) भधित + ढहंद ू
(ख) भधित + ढहंद ु
(ग) भधित + इंद ू
(घ) भधित + इंद ु

(ii). पौ+अक की संचध होगी—


(क) पधवक
(ख) पौअक
(ग) पोवक
(घ) पवक

(iii). निम्िलिखखत में से सही संचध ववच्छे र्द वािा ववकल्प चुनिए—
(क) स + दै व
(ख) महध + ऋवि
(ग) प + वि
(घ) भ + वि

(iv). निम्िलिखखत यग्ु मों पर ववचार कीक्जए—


संचध संचध ववच्छे र्द
(i) सूयोदय (i) सूिि + उदय
(ii) वपत्रधदे श (ii) वपत ृ + आदे श
(iii) स्वधगत (iii) स्व + आगत
(iv) िधिेंद्र (iv) िधिध + इन्द्र

उपयभत
ु त यग्ु मों में से कौि से सही सम
ु ेलित हैं?
(क) (i) औि (ii) (ख) (i) औि (iii)
(ग) (iii) औि (iv) (घ) (ii) औि (iv)
7. निम्िलिखखत प्रश्िों के सही उत्तर दर्दए गए ववकल्पों में से
चनु िए— (1×3=3)

(i) सोिजुही में आज एक पीिी किी िगी है । वातय में उचचत ववराम चचह्ि होगा—
(क) ववस्मयधढदबोर्क
(ख) प्रश्ि सच
ू क
(ग) पूर्ावविधम
(घ) योिक चचह्ि

(ii). निम्िलिखखत में से ककस में उचचत ववराम चचह्ि िगा है —


(क) एमए. बीएड.
(ख) एम. ए. ; बी.एड.
(ग) एम. ए. बीएड.
(घ) एमए. बी.एड.

(iii). हे र्गवाि तुम्हें कब अति आएगी - वातय में उचचत ववराम चचह्ि होगा—
(क) पूर्ावविधम तथध प्रश्ि सूचक
(ख) ववस्मयधढदबोर्क एवं प्रश्ि सूचक
(ग) लधघव चचह्ि तथध पूर्ावविधम
(घ) अकपवविधम तथध ववस्मयधढदबोर्क

(iv).ककसी के उपिाम, कववता, पस्


ु तक के शीषभक आदर्द को उदधत
ृ करते समय
ककस चचह्ि का प्रयोग ककया जाता है —
(क) दह
ु िध उद्र्िर् चचह्ि
(ख) योिक चचह्ि
(ग) निदे शक चचह्ि
(घ) इकहिध उद्र्िर् चचह्ि

8.निम्िलिखखत प्रश्िों में से ककन्हीं 2 प्रश्िों के उत्तर


र्दीक्जए— ((1×2=2)
(i).यदर्द तुम कि घर जाओगे तो मैं र्ी तुम्हारे साथ चिँ ग
ू ा। वातय में अथभ की
दृक्ष्ट से वातय-र्ेर्द होगा—
(क) ववस्मयवधचक (ख) संकेतवधचक
(ग) संदेहवधचक (घ) आज्ञधवधचक

(ii).अथभ की दृक्ष्ट से वातय-र्ेर्द बताइए—


अहा! इतिा सुंर्दर फूि र्दे खकर मि प्रसन्ि हो उठा।
(क) संदेहवधचक (ख) इच्छधवधचक
(ग) ववर्धिवधचक (घ) ववस्मयवधचक

(iii). हम तुम्हें स्टे शि तक छोडिे जाएँगे। वातय का िकारात्मक/ निषेधात्मक


वातय होगा—
(क) हम तम्
ु हें स्टे शि तक छोडिे िह ं िधएँगे।
(ख) क्यध हम तुम्हें स्टे शि तक छोडिे िधएँगे?
(ग) क्यध हमें तुम्हें स्टे शि तक छोडिे िधिध होगध?
(घ) हमें तम्
ु हें स्टे शि तक क्यों छोडिध चधढहए?

9. निम्िलिखखत पदयांश को पढ़कर पछ


ू े गए प्रश्िों के उत्तर के लिए सही ववकल्प का चयि
कीक्जए—. (1x5=5)
यहधँ िोज कुछ बि िहध है
िोज कुछ घट िहध है
यहधँ स्मनृ त कध भिोसध िह ं
एक ह ढदि में पुिधिी पड िधती है दनु ियधँ
िैसे वसंत कध गयध पतझड को लौटध हूँ
िैसे बैसधख कध गयध भधदों को लौटध हूँ
अब यह है उपधय कक हि दिवधजध खटखटधओ
औि पूछो— क्यध यह है वो घि?

समय बहुत कम है तुम्हधिे पधस


आ चलध पधिी ढहध आ िहध अकधस
शधयद पक
ु धि ले कोई पहचधिध ऊपि से दे खकि
(I) कवव को स्मनृ त पर र्रोसा तयों िहीं है?
(क) पिु धिे िधस्तों पि भिोसध िह ं
(ख) पिु धिी यधदों की निशधनियधँ बदल गईं हैं
(ग) पिु धिे लोग बदल गए
(घ) उि सब कध स्वभधव बदल गयध
(ii) र्दनु िया एक ही दर्दि में परु ािी तयों पड जाती है?
(क) िो कल थध वह आि िह ं
(ख) बीतध समय कभी वधवपस िह ं आतध
(ग) भत
ू कधल को समझिे के कधिर्
(घ) नित-िए परिवतािों के कधिर्
(iii) ‘जैसे वसंत का गया पतझड को िौटा हूँ’ से कवव तया कहिा चाहता है?
(क) ऋतुओं कध बदल िधिध
(ख) पत्तों कध झड िधिध
(ग समय परिवतािशील है
(घ) सुख कध दख
ु ों में बदलिध
(iv) ‘शायर्द पक
ु ार िे कोई पहचािा ऊपर से र्दे ख कर’ पंक्तत कवव के ककस र्ाव को प्रकट
करती है?
(क) उसे कोई पुकधििध चधहतध है
(ख) िकधिधत्मक दृजष्ट्टकोर्
(ग) सकधिधत्मक भधव
(घ) वह थक गयध है
(v) उपयुभतत काव्यांश के संबध
ं में कौि-सा कथि असत्य है?
(क) यहधँ हि ढदि कुछ ि कुछ ियध निमधार् होतध िहतध है
(ख) निमधार् व वविधश की प्रकक्रयध चलती िहती है।
(ग) यहधँ सब कुछ अस्थधयी है।
(घ) कवव को अपिे घि की तलधश है।

10.निम्िलिखखत प्रश्िों में से सही उत्तर वािे ववकल्पों को चुनिए—. (1x2=2))


(I) िर्दी ककसका गीत गा रही है?
(क) खश
ु ी कध
(ख) लमलि कध
(ग) वविह कध
(घ) उमंग कध
(ii) िर्दी अपिे मि की व्यथा ककससे कहती है?
(क) सधगि से
(ख) ककिधिों से
(ग) पेड-पौर्ों से
(घ) गल
ु धब से

11. निम्िलिखखत गदयांश को पढ़कर पछ


ू े गए प्रश्िों के सही उत्तर ववकल्पों से
चनु िए—. (1x5=5)

उस ढदि िब तम
ु आए थे, मेिध ह्रदय ककसी अज्ञधत आशंकध से र्डक उठध थध। अंदि ह अंदि
कह ं मेिध बटुआ कधँप गयध। उसके बधविूद एक स्िेह-भीगी मुसकिधहट के सधथ मैं तुमसे गले
लमलध थध औि मेि पत्िी िे तुम्हें सधदि िमस्ते की थी। तुम्हधिे सम्मधि में ओ अनतचथ,
हमिे िधत के भोिि को एकधएक उच्च-मध्यम वगा के डडिि में बदल ढदयध थध । तुम्हें
स्मिर् होगध कक दो सजब्जयों औि िधयते के अलधवध हमिे मीठध भी बिधयध थध। इस सधिे
उत्सधह औि लगि के मूल में एक आशध थी कक दस
ू िे ढदि ककसी िे ल से एक शधिदधि
मेहमधििवधजी की छधप अपिे ह्रदय में ले तुम चले िधओगे। हम तुमसे रुकिे के ललए आग्रह
किें गे, मगि तुम िह ं मधिोगे औि एक अच्छे अनतचथ की तिह चले िधओगे। पि ऐसध िह ं
हुआ ; दस
ू िे ढदि भी तुम अपिी अनतचथ सुलभ मुस्कधि ललए घि में ह बिे िहे । हमिे
अपिी पीडध पी ल औि प्रसन्ि बिे िहे । स्वधगत-सत्कधि के जिस उच्च बबंद ु पि हम तुम्हें
ले िध चक
ु े थे ,वहधँ से िीचे उति हमिे कफि दोपहि के भोिि को लंच की गरिमध प्रदधि की
औि िधबत्र को तुम्हें लसिेमध ढदखधयध। हमधिे सत्कधि कध यह आखखि छोि है, जिससे आगे हम
ककसी के ललए िह ं ब़िे ।
(I) अनतचथ के आगमि का िेखक पर तया प्रर्ाव पडा?
(क) उन्हें बहुत अच्छध लगध
(ख) सत्कधि में िटु गए
(ग) ककसी अज्ञधत आशंकध से कधँप उठे
(घ) बहुत प्रसन्ितध हुई
(ii) अनतचथ के आगमि पर उिका स्वागत ककस प्रकार ककया गया? दर्दए गए कथिों पर
ववचार कीक्जए और उचचत ववकल्प का चयि कीक्जए।
(I)लेखक मस
ु किधहट के सधथ अनतचथ से गले लमलध
(ii) लेखक की पत्िी िे सधदि िमस्ते की
(iii) खधिे में दो सजब्जयों के सधथ िधयते को भी शधलमल ककयध गयध
(iv) लेखक अपिे सधथ उन्हें घम
ु धिे ले गए
(क) (i) औि (ii)
(ख) (i), (ii) औि (iii)
(ग) केवल (iv)
(घ) (i), (iii) औि (iv)
(iii) िेखक िे रात के र्ोजि को ककस रूप में बर्दि दर्दया?
(क) निम्ि वगा के डडिि में बदल ढदयध
(ख) उच्च-मध्यम वगा के डडिि में बदल ढदयध
(ग) मध्यम वगा के डडिि में बदल ढदयध है
(घ) उच्च वगा के डडिि में बदल ढदयध
(iv) िेखक के अिस
ु ार अच्छे अनतचथ की तया पहचाि होती है?
(क) चधि-पधँच ढदि िह िधतध है ।
(ख) िो खधिे को दो, चुपचधप खध लेतध है।
(ग) घि के कधयों में सहधयक बितध है ।
(घ) एक शधिदधि मेहमधििवधजी की छधप लेकि अगले ढदि चलध िधतध है।
(v) कथि (A): िेखक िे रात के र्ोजि को एकाएक उच्च-मध्यम वगभ के डडिर में बर्दि
दर्दया था।
कारण (R): िेखक िे अनतचथ की शािर्दार मेहमाििवाजी की।
(क) कथि (A) तथध कधिर् (R) दोिों ग़लत हैं।
(ख) कथि (A) ग़लत हैं ,लेककि कधिर् (R) सह है।
(ग) कथि (A) सह है, लेककि, कधिर् (R) ग़लत व्यधख्यध कितध है।
(घ) कथि (A) तथध कधिर् (R) दोिों सह है तथध कधिर् (R) कथि (A) की सह व्यधख्यध
कितध है ।

12. निम्िलिखखत प्रश्िों में से सही उत्तर ववकल्पों से चुनिए— (1x2=2)

(I) बर्फभ के खंडों का अव्यवक्स्थत ढं ग से चगरिा तया कहिाता है?


(क) ढहमपधत
(ख) खंडडत बफ़ा
(ग) खंडडत चट्टधिें
(घ) खंडडत पवात
(ii) रसोई सहायक की मत्ृ यु का तया कारण था?
(क) िलवधयु कध अत्यचर्क गमा होिध
(ख) िलवधयु कध अिक
ु ू ल ि होिध
(ग) ग्लेलशयि कध वपघलिध
(घ) पवात कध खखसकिध

खंड-‘ब’ (वणभिात्मक प्रश्ि)


13. निम्िलिखखत में से ककन्हीं र्दो प्रश्िों के उत्तर िगर्ग 60 शब्र्दों में
लिखखए— (3x2=6)

(I) सि चंद्रशेखि वेंकटिधमि की खोि भौनतकी के क्षेत्र में एक क्रधंनत के समधि थी।
इस कथि पि अपिे ववचधि प्रकट कीजिए ।

(ii) महधदे व भधई दे सधई की तुलिध शुक्रतधिे से क्यों की गई है ? महधत्मध गधंर्ी के


िीवि में महधदे व भधई दे सधई कध क्यध स्थधि थध?

(iii) कहधिीकधि यशपधल िे ‘दख


ु कध अचर्कधि’ कहधिी में ककस सधमधजिक समस्यध
पि प्रकधश डधलध है ?

14. निम्िलिखखत में से ककन्हीं र्दो प्रश्िों के उत्तर िगर्ग 60 शब्र्दों में लिखखए
लिखखए— (3x2=6)

(I)’सब सधर्े सब िधय’ पंजक्त में िह म िे मधिव स्वभधव की ककस ववशेितध की


ओि संकेत ककयध है ?

(ii) आपके पधठ्यक्रम की कववतध ‘अजग्िपथ’ में मधिव िीवि को अंगधिों भिे िधस्ते
के समधि बतधयध गयध है । ऐसे िधस्तों पि नििं ति आगे ब़िते िहिे के ललए हमें
क्यध कििध चधढहए? संकट की जस्थनत में क्यध दस
ू िों से मदद लेिध सह है ?

(iii) ‘खश
ु बू िचते हैं हधथ’ कववतध के आर्धि पि स्पष्ट्ट कीजिए कक ककस प्रकधि
दख
ु -ददा में सुख औि आिंद तथध दग
ु ांर् के बीच सुगंर् कध सि
ृ ि होतध है ?

15. निम्िलिखखत में से ककन्हीं र्दो प्रश्िों के उत्तर िगर्ग 60 शब्र्दों में
लिखखए— (3x2=6)

(I)चगलहि के बच्चे कध उपचधि लेखखकध िे ककस प्रकधि ककयध?


(ii) ट ललयधमिु ध कस्बे में लेखक कध परिचय ककि दो प्रमख
ु हजस्तयों से हुआ?
समधि ककयधर् के कधयों में उिकध क्यध योगदधि थध?

(iii) लेखक को ककतधब प़ििे एवं सहे ििे कध शौक़ कैसे लगध? ‘मेिध छोटध सध नििी
पुस्तकधलय’ पधठ के आर्धि पि ललखखए।

रचिात्मक िेखि
16.दर्दए गए संकेत बबंर्दओ
ु ं के आधार पर ककसी एक ववषय पर िगर्ग 100 शब्र्दों
में अिुच्छे र्द लिखखए— (6)
(i) स्वच्छ र्ारत उन्ित र्ारत
संकेत बबंद ु
- प्रधचीि भधित में स्वच्छतध की ककपिध
- स्वच्छतध िीवि कध आर्धि
- स्वच्छतध है तो ववकधस है

(ii) जि ही जीवि है
संकेत बबंद ु
- पधिी तेिे रूप अिेक
- पधिी िीवि कध अनिवधया तत्व
- पधिी की भववष्ट्य में कमी
- िल संिक्षर् औि शुद्चर्किर्

(iii) युदध शांनत का अंनतम उपाय िहीं


संकेत बबंद ु
- युद्र् के कधिर्
- युद्र् से वविधश
- शधंनत पथ ह अंनतम ववककप

17. र्दार्दा-र्दार्दी के साथ रहिा आपके लिए वरर्दाि लसदध हो रहा है - प्रमाखणत करते
हुए लमत्र को िगर्ग 100 शब्र्दों में पत्र लिखखए । (6)
अथवध

“स्वस्थ रहिे के लिए व्यायाम करिा ककतिा आवश्यक है ।”- छोटी बहि को
िगर्ग 100 शब्र्दों में पत्र लिखकर समझाइए।

18.वित्र में वदखाई दे र े दृश्य/ घटना का अपनी कल्पना शवि से वित्र वर्ण न लगभग 100 शब्दों में कीविए-(5)

19. बहुत दर्दिों के बार्द लमत्र से र्ेंट होिे पर र्दो लमत्रों के मध्य होिे वािे संवार्द
को िगर्ग 100 शब्र्दों में लिखखए- (5)

अथवा

जेब खचभ बढ़ािे के संर्दर्भ में माँ - पुत्र /पुत्री के मध्य होिे वािे संवार्द को िगर्ग
100 शब्र्दों में लिखखए।
नमूना प्रश्न-पत्र 2023-24
विषय- हिन्दी (कोर्स-बी)
उत्तर र्ंकेत
कक्षा- निीं

समयः 3 घंटे पूर्णांक-80


1. अपहित गदयांश (1x5=5)
i.(ख) जीवन के अनुभवों पर ।
ii..(क) पणररवणररक सदस्य यण ककसी अन्य को अपनण आदर्श बनणकर।
iii..(घ) उपयुशक्त में से कोई नह ं।
iv.(ख) थकणवट, अज्ञणत डर,च त
ं णओं,झुुँझलणहट से।
v. (घ) कथन (A) तथण कणरर् (R) दोनों सह है तथण कणरर् (R) कथन (A) की सह व्यणख्यण करतण है ।
2. अपहित गदयांश (1x5=5)
i. (ग) ततरंगे कण रं ग,आकणर,बनणवट,रं गों कण क्रम आदद के तनर्णशरर् को।
ii.(घ) सत्य एवं र्मश ह मनुष्यतण के तनयणमक ससदर्णंत हैं ।
iii.(ख) इसकण अनुपणत 2 : 3 होनण णदहए।
iv.(घ) रणष्र यतण की।
v.(घ) कथन (A) तथण कणरर् (R) दोनों सह हैं तथण कणरर् (R) कथन (A) की सह व्यणख्यण करतण है ।

व्याििाररक व्याकरण

3. i (ग) बेटण (1×2=2)

ii (ख) पद

4
I. (क) हंस (1×2=2)

ii. (क) कणुँ

iii. (ख) कं न

5.
i (क) अचर् (1×4=4)

ii. ( ख) दुबशल

iii (ग) तण

iv (घ) उदणर

v. (ग) ई

6
i (घ) भणरत + इंदु (1×3=3)

ii. (क) पणवक

iii (ख) महण + ऋषि

iv (घ) (ii). और (iv)

7.
i (ग) पूर्शषवरणम (1×3=3)

ii. (ख) एम.ए.; बी.एड.

iii. (ख) षवस्मयणददबोर्क तथण प्रश्न सू क

iv. (घ) इकहरण उदर्रर् च ह्न

8.

i. ( ख) संकेतवण क (1×2=2)

ii. (घ) षवस्मयवण क

iii (क) हम तुम्हें स्टे र्न तक छोड़ने नह ं जणएुँगे।


पाठ् यपस्ु तक स्पर्श भाग-1कणव्य खंड
पहित गदयांश पर एक अंकीय पााँच बिुविकल्पी प्रश्न(1x5=5)
9.
I.(ख) पुरणनी यणदों की तनर्णतनयणुँ बदल गई हैं ।
II.(घ) तनत-नए पररवतशनों के कणरर् ।
III.(ग) समय पररवतशनर्ील है ।
IV.(ग) सकणरणत्मक भणव।
V.(घ) कषव को अपने घर की तलणर् है ।
स्पशस (भाग-1) र्े ननर्ासररत कविताओं के आर्ार पर एक अंकीय दो बिुविकल्पी प्रश्न (1x2=2)
10.
I (ग) षवरह कण
ii (ख) ककनणरों से
गदय खंड
पहित गदयांश पर एक अंकीय पााँच बिु विकल्पी प्रश्न (1x5=5)
11.
I. (ग) ककसी अज्ञणत आर्ंकण से कणुँप उठे ।
ii. (क) (I) और (ii)
iii. (ख) उच् मध्यम वगश के डडनर में बदल ददयण।
iv. (घ) एक र्णनदणर मेहमणननवणजी की छणप लेकर अगले ददन लण जणतण है ।
v. (घ) कथन (A) तथण कणरर् (R) दोनों सह हैं तथण कणरर् (R) कथन (A) की सह व्यणख्यण करतण है ।
स्पशस भाग-1 र्े ननर्ासररत गदय पािों के आर्ार एक अंकीय दो बिु विकल्पी प्रश्न । (1x2=2)
12.
i. (क) दहमपणत
ii. (ख) जलवणयु कण अनुकूल न होनण।
खंड-‘ब’ िणसनात्मक प्रश्न
पाठ्यपुस्तक स्पशस भाग-1 (गदय खंड) र्े ननर्ासररत पािों के आर्ार पर तीन में र्े दो प्रश्न । (3x2=6)
13.
i.
. आइंस्टणइन के षव णरों कण प्रणयोचगक प्रमणर् समलनण।
. रणमन ् स्पेक्रोस्कोपी पदणथों के अर्ुओं और परमणर्ुओं की आंतररक संर नण को समझने में सहणयक।
. वैज्ञणतनक दृष्ष्ट और च त
ं न में सहणयक।
ii.
. महणदे व भणई दे सणई कण व्यष्क्तत्व र्ुक्रतणरे के समणन दे द प्यमणन ।
. दे र् और दुतनयण को मुग्र् करके र्ुक्र तणरे के समणन है अ णनक अस्त हो जणनण।
. महणत्मण गणंर्ी के षवसर्ष्ट सहयोगी। उनकी तमणम इतर च त
ं णओं को अपने ससर लेने वणले ।
. महणत्मण गणंर्ी दवणरण वणररस घोषित ककयण जणनण।
iii.
. र्नी लोगों की अमणनवीयतण व गर बों की मजबूर को उजणगर करनण।
. समणज में फैले अंर्षवश्वणसों पर णश।
. दुख कण अचर्कणर से ग़र ब इंसणन वंच त।
पाठ्यपुस्तक स्पशस भाग-1 (पदय खंड) र्े ननर्ासररत पािों के आर्ार पर तीन में र्े दो प्रश्न । (3x2=6)
14.
i.
. मणनव परमणत्मण को षवस्मत
ृ कर दे तण है ।
. सणंसणररक सुखों में भटक जणतण है ।
. पररर्णमतः णरों ओर ध्यणन भटकणने पर ककसी भी लक्ष्य तक नह ं पहुुँ ण जण सकतण।
ii.
. जीवन की षवसभन्न समस्यणओं से जूझनण । अपने पथ पर तनरं तर बढ़ते रहनण ह मणनव र्मश है ।
. छणयण अथणशत सुख की कणमनण न करनण।
. कदठन पररष्स्थततयों में सहणयतण मणुँगनण अनुच त क्योंकक इससे आत्मषवश्वणस डगमगणतण है और उत्सणह में भी कमी आतीहै ।
iii.
. उपेक्षण, र्णर ररक पीड़ण व दुगांर्पूर्श वणतणवरर् में भी अपने पररवणर कण भरर्-पोिर् करने के सलए षववर्।
. खुर्बूदणर अगरबषियणुँ बनणनण और हमणरे जीवन को सुगंर् और सुखद अनुभूत त से भरनण।
. पीड़ण में ह मु सकरणहट और प्रसन्नतण बबखेरकर , दुगांर् के बी ह सुगंर् कण सज
ृ न।

पूरक पाठ्यपुस्तक र्ंचयन ननर्ासररत बातों र्े पािों में र्े तीन में र्े दो प्रश्न (3x2=6)
15.
I.
. रक्त पोंछकर घणवों पर पें सससलन कण मरहम ।
. रुई की पतल बिी दूर् में सभगोकर ,दूर् की बूं दे मुुँह में डणलने कण प्रयणस।
. लेखखकण के अथक प्रयणसों से तीन ददन में पूर तरह स्वस्थ।
ii.
. लोक गणयक हे मंत कुमणर जमणततयण और मंजु ऋषिदणस से ।
. ष्जलण पररिद कण सदस्य बनकर जनसेवण कण दणतयत्व।
. वणडश के सलए पं णयत की ओर से स्वच्छ पेयजल और मुख्य गसलयों में ईंटें बबछवणने की व्यवस्थण।
iii.
. ब पन में बणल पबिकणओं को तनयसमत रूप से पढ़नण।
. स्वणमी दयणनंद सरस्वती के जीवन की रोमणं क घटनणओं से प्रभणषवत ।
. पणं वीं कक्षण में प्रथम आने पर स्कूल से इनणम में समल दो अंग्रेजी ककतणबें । षपतण के दवणरण अलमणर के खणने में उनकी दोनों ककतणबों
कण रखनण तथण कहनण आज से यह तुम्हणर अपनी लणइब्रेर है ।

16. अनुच्छे द (6×1=6)

षवियवस्तु 3 अंक

भणिण 2 अंक

प्रस्तुतत 1अंक

17. अनौपचाररक पत्र (6×1=6)

आरम्भ की औप णररकतणएुँ 1 अंक

षवियवस्तु 3 अंक

भणिण 1 अंक

प्रस्तुतत 1 अंक

18. चचत्र िणसन (5×1=5)

षवियवस्तु 3 अंक

भणिण 1 अंक
प्रस्तुतत 1अंक

19. र्ंिाद लेखन (5×1=5)

षवियवस्तु 3 अंक

भणिण 1 अंक

प्रस्तुतत 1अंक
BLUE PRINT CLASS IX MATHS 2023-2024 (ANNUAL)

NAME OF MCQ’S ASSERTION SA I SA II LA CASE BASED TOTAL


THE REASON Q’S QUESTIONS
CHAPTER 1 MARK 1 MARK 2 MARKS 3 MARKS 5 MARKS 4 MARKS
Number 2 1 1 10(4)
Systems

Algebra 4 1 3 1 1 20(10)

Coordinate 3 1 4(4)
Geometry

Geometry 5 2 3 1 1 27(12)

Mensuration 3 2 1 13(6)

Statistics 1 1 6(2)

TOTAL 18(18) 2(2) 10(5) 18(6) 20(4) 12(3) 80(38)


Class- IX Session- 2023-24
Subject- Mathematics
Sample Question Paper(Annual)
Time Allowed: 3 Hrs. Maximum Marks : 80
General Instructions:
1. This Question Paper has 5 Sections A-E.
2. Section A has 20 MCQs carrying 01 mark each
3. Section B has 5 questions carrying 02 marks each.
4. Section C has 6 questions carrying 03 marks each.
5. Section D has 4 questions carrying 05 marks each.
6. Section E has 3 case based integrated units of assessment (04 marks
each) with sub-parts of the values of 1, 1 and 2 marks each
respectively.
7. All Questions are compulsory. However, an internal choice of 2 Qs
of 5 marks, 2 Qs of 3 marks and 2 Questions of 2 marks has been
provided. An internal choice has been provided in the 2 marks
questions of Section E
8. Draw neat figures wherever required. Take π =22/7 wherever
required if not stated.

SECTION A
So.No SECTION A consists of 20 Questions of 1 Mark each Marks
.

1. The width of each of five continuous classes in a frequency 1


distribution is 5 and the lower class limit of the lowest class is 10.
The lower class limit of the highest class is
(a) 25 (b) 30 (c) 35 (d)40
2. Which of the following Euclid’s postulates, ensures the existence of 1
parallel lines?
(a) Postulate 1 (b) Postulate 2 (c) Postulate 4 (d) Postulate 5
An isosceles right triangle has an area of 8 sq.cm.The length of its 1
3. hypotenuse is
(a) √32 cm (b) √16 cm (c) √48 cm (d) √24 cm

The three vertices of ∆XYZ are X(1,4),Y(-2,2) and Z(3,2).The area of 1


4. ∆XYZ in sq.units is
(a) 4 (b) 5 (c) 10 (d) 20
In the
5. In the given figure, AB || CD then value of x is 1

(a) 45° (b)60° (c) 90° (d)105°

6. Which of the following is true for multiplication of two irrational 1


numbers?
(a)Always a real number (b)Rational or Irrational
(c ) Always irrational (d)Both (a) and (b)
7. The difference between the degree of a quadratic polynomial and 1
the number of terms in a trinomial is
(a) -2 (b) 1 (c ) −1 (d) 2
8. If AB = QR, BC = RP and CA = PQ, then 1
(a) ∆ ≅∆ (b) ∆ ≅∆
(c) ∆ ≅∆ (d) ∆ ≅∆
9. If (x +1) is a factor of ax³+x²-2x + 4a -9,then the value of a is 1
(a)2 (b) 1 (c )-2 (d) -1
10. The figure obtained by plotting the points (2, 3) , (-2,3) , (-2,-3) and 1
(2, -3) is a
(a)Trapezium (b) Rectangle (c )Square (d)Rhombus

The coordinates of the point which lies on x-axis at a distance of 6 1


11. units in negative direction of x-axis is
a) (5 (a) (0,6) (b) (6, 0) (c) (0, -6) (d) (-6, 0)

If one angle of a parallelogram is 36° less than twice its adjacent angle, then 1
12. find the largest angle of the parallelogram.
(a) 108° (b) 72° (c ) 36° (d)120°
The radius of a hemispherical balloon increases from 6 cm to 12 cm 1
13 . as air is being pumped into it. The ratios of the surface areas of the
balloon in the two cases is
(a) 1: 4 (b) 1: 3 (c) 2: 3 (d) 2: 1

If the linear equation has solutions(-2,2),(0,0),(5,-5),then the 1


14. equation is
(a)x - y = 0 (b) x +2y = 0 (c ) 2x - y =0 (d) x + y = 0
In the figure O is the center of the circle , what is the measure of 1
15. angle ACB

(a) 45° (b)60° (c ) 70° (d) 90°

The mirror image of point (3,4) in x-axis has the coordinate 1


16. (a) (-3,4) (b) (3,-4) (c) (-3,-4) (d) (4,3)
/
The Product of √ . . √ equals 1
17.
a) ) √ b) √ c) √ d) 2

A cone is 8.4 cm high and the radius of its base is 2.1 cm. It is 1
18. melted and recast into a shape. The radius of the sphere is
(a) 4.2 cm (b) 2.1 cm. (c) 2.4 cm (d) 1.6 cm
DIRECTION: In question number 19 and 20, a statement of assertion 1
19. (A) is followed by a statement of Reason (R).
Choose the correct option

Assertion(A): In 3rd quadrant every ordered pair(x,y) consists of two


negative numbers.
Reason (R ): The axes divide the plane into four regions called
quadrants, which are numbered counter clockwise.

(a)Both A and R are true and R is the correct explanation of A.


(b)Both A and R are true but R is not the correct explanation of A
(c ) A is true but R is false.
(d)A is false but R is true.

Assertion(A): If the sum of all the coefficients,including the constant 1


20. term,of a polynomial is zero,then (x-1) is one of its factors.
Reason (R): If a polynomial f(x) is divisible by (x-t),then f(t)=0.

(a)Both A and R are true and R is the correct explanation of A.


(b)Both A and R are true but R is not the correct explanation of A
(c )A is true but R is false.
(d)A is false but R is true.

SECTION B
So.no. Section B consists of 5 questions of 2 marks each. Marks
21. In the given figure QP ∥ LM, find the value of x. 2

OR
In the given figure, lines AB, CD and EF intersect at O.
Find the measure of ∠AOC and ∠COF.

22. A chord of a circle is 16 cm in length and its distance from the 2


center is 6 cm. Find the length of the chord of the same circle
at a distance of 8 cm from the center.
23. If x + y = 12 and xy = 27 , find the value of 2
+
OR

Factorise :
4√ + 5x - 2 √

24. 2
If the point (2k-3,k+2) lies on the graph of the equation 2x
+ 3y +15 = 0, find the value of k.

25. What could be the possible expressions for the length and 2
breadth of the rectangle whose area is given by (25a2 − 35a + 12)
m2 ?

SECTION C
So.No. Section C consists of 6 questions of 3 marks each Marks
26 Find the value of a and b in + =a√ + √ 3
√ √ √ √

27 In the given figure, POQ is a line. OS is another ray lying 3


between OR and OP.
Prove that: ∠ROS = (∠QOS -∠POS).
28 3
AB is a line segment and P is its midpoint. D and E are points on
the same side of AB such that ∠BAD = ∠ABE and ∠EPA = ∠DPB.

Show that

(i) ΔDAP ≅ ΔEBP

(ii) AD = BE

Or
In the given figure, D and E are the points on the base BC of △ABC
such that BD=CE, AD = AE and ∠ADE = ∠AED. Prove that △ABE ≅
△ACD.
29 If two parallel lines are intersected by a transversal, then prove 3
that bisectors of the interior angles form a rectangle.

30 A semicircular sheet of metal of diameter 28cm is bent into an 3


open conical cup. Find depth and capacity of the cup.
Or
If the radius of a sphere is doubled, find the percentage increase
in its total surface area.
31 The perimeter of a triangle is 50cm.One side of the triangle is 4 3
cm longer than the smaller side and the third side is 6 cm less
than twice the smaller side.Find the area of the triangle.

SECTION D
So.No. Section D consists of 4 questions of 5 marks each Marks
32 Prove that angle subtended by an arc of a circle at the center is 5
double the angle subtended by it on the remaining part of the
circle.

During a practical activity in the Math lab, students were using a


circular geoboard. The angle subtended by an arc at the center
was (3a + 15°). Pallavi calculated ∠BAC as (a + 25°).Find the
measure of ∠BAC

33. If f(x) = x⁴-2x³+3x²-ax+b is a polynomial such that when it is 5


divided by( x -1)and (x+1), the remainders are respectively 5 and
19. Determine the remainder when f(x) is divided by( x - 2)
Or
a) Factorise: 16x² + 4y² + 9z² – 16xy – 12yz + 24xz
b) If p +q+ r = 9, find the value of
3 3 3
(3 − ) + (3 − ) +(3 − !)
34 If 5
" # − $√% = √ − √ then Prove that a+b=18.

OR

If x= , find the value of 4x3+2x2-8x+7.

35 Draw histogram and frequency polygon for the following 5


distribution table.

Marks No. Of students


600-640 16
640-680 45
680-720 156
720-760 284
760-800 172
800-840 59
840-880 18

SECTION E
Case study based questions are compulsory
36 Gaming zones are a popular concept in the gaming industry.
They are typically areas within a gaming establishment where
gamers can play video games without having to worry about
other people or the environment. This can be a great place to
relax and have some fun, while also getting some exercise
The brainstorming games are quite helpful for the mental
growth of the kids. An hourly fee is typically charged in
gaming areas. The lowest price point is ₹ 50 per hour. High-
end gaming areas often charge between ₹ 200 and 300 per
hour .
The funky island ,a gaming zone charges ₹200 for one hour
and ₹75 for subsequent hours played.

Refer the given information of funky island and answer the


following questions by taking total hours spend x hours and
total amount paid ₹y
(i) Write the linear equation for above mentioned information.
(ii)How many solutions are there for linear equation representing
the above situation.
(iii) If a child spends 3 hours in funky island. Find the amount
paid by him.
Or
Express the linear equation obtained in (i) in the form of
1
ax +by +c = 0 and indicate the values of a,b and c.
1

2
37 Case Study: Radha Krishnan's Cone Business

Radha Krishnan, an unemployed individual, decided to venture


into a small business to make ends meet. His entrepreneurial
spirit led him to the world of street food, specifically selling bhel
and chana jor garam. To present these snacks in a convenient
and attractive way, Radha Krishnan chose to prepare cones
made of paper.
Cone Specifications:
- Height of each cone: 21cm
- Slant height of each cone: 28cm
This case study illustrates the practical application of geometry
in a small business setting, emphasizing the importance of
precise measurements for cost-effective production.
Read the above situation and answer the following questions
1
(i) Find radius of cone so formed.
1
(ii) How much paper does he require to make one such cone. 2
(iii) Find the volume of 10 such cones.
Or
If the radius and slant height of a cone are in the ratio of 3:5,its
new curved surface area is 2310cm sq.then find its new radius
and height.
38 A resident welfare society, seeking to enhance community
living, embarked on the development of a unique land parcel
with a parallelogram shape measuring 120 by 100 meters. The
society aimed to create a cohesive environment by introducing a
new quadrilateral formed by joining the midpoints of
consecutive sides in the middle of the land.

The primary objective of this case study is to explore how the


strategic development of the land, including the creation of a
central quadrilateral, contributes to fostering community
engagement, aesthetics, and functionality within the resident
welfare society.

Read the above information and answer the following question


1
1. What is the shape of quadrilateral PQRS formed? 1
2. If the length of PQ is 90 m. What is the length of AC? 2
3. The society wants to fence the land. If the cost of fencing
1m is ₹ 75, then find out the cost of fencing the whole
land.Also Find the area of triangle PQB?
OR
Diagonals AC and BD of a parallelogram ABCD intersect
each other at O, if OA=50m and OD =30 m determine the
length of AC and BD. Also find ∠A -∠C.
Class : IX Science

SAMPLE PAPER ( 2023- 24 )


Blue Print

PHYSICS - 27 marks
CHEMISTRY - 25 marks
BIOLOGY – 28 marks

S.No
Name of the MCQ, Very Short Long Case Total
chapter Assertion-Reason Short Answer Answer study
Type Questions Answer type Type type
type Question Questions Question
(1 mark) Question (3 marks)
(5 marks) (4 marks)
(2 marks)

1. Motion 1(1) MCQ 1(4) 2(5)

2. Force and 1(2) 1(3) 2(5)


Laws of
Motion

3. Gravitation 1(2) 1(3) 2(5)

4 Work and 1(1) MCQ 1(5) 2(6)


Energy

5 Sound 1(1) A/R 1(2) 1(3) 3(6)


6 Fundamental 3(1) MCQ 1(2) 1(3) 5(8)
Unit of Life

7 Tissues 3(1) MCQ 1(3) 1(5) 6(12)

1(1) A/R

8 Improvement 1(1) MCQ 1(2) 1(4) 4(8)


in Food 1(1) A/R
Resources

10

11

12 Matter in our 2(1)MCQ 1(3) 3(5)


surroundings

13 Is matter 2(1)MCQ 4(5)


around us 1(1)A/R 1(2)
pure

14 Atoms and 1(3) 1(5) 2(8)


Molecules
15 Structure of 1(4) 4(7)
3(1) MCQ
the atom

TOTAL 20(1) 6(2) 7(3) 3(5) 3(4) 39


(80)

Note: ( ) denotes marks.

AR means Assertion Reason Type Question

Total number of questions is 39

Q37 to Q39 are Case based questions, containing two or three sub-parts (with internal
choice in one of these sub-parts)
Science (086)
Class IX
SAMPLE PAPER 2023-24
Max. Marks: 80 Time Allowed: 3 hours

General Instructions:
i. This question paper consists of 39 questions in 5 sections.
ii. All questions are compulsory. However, an internal choice is provided in some
questions. A student is expected to attempt only one of these questions.
iii. Section A consists of 20 objective type questions carrying 1 mark each.
iv. Section B consists of 6 Very Short questions carrying 02 marks each. Answers to
these questions should be in the range of 30 to 50 words.
v. Section C consists of 7 Short Answer type questions carrying 03 marks each.
Answers to these questions should be in the range of 50 to 80 words.
vi. Section D consists of 3 Long Answer type questions carrying 05 marks each.
Answer to these questions should be in the range of 80 to 120 words.
vii. Section E consists of 3 source-based/case-based units of assessment of 04
marks each with sub-parts

S. Questions Marks
No.

Section-A
Select and write the most appropriate option out of the four
options given for each of the questions 1 - 20. There is no
negative mark for incorrect response.

1. Aditya visited a natural gas compressing unit and found that the 1
gas can be liquified under specific conditions of temperature and
pressure. Identify the correct set of conditions used to liquify the
gas.
(a) Low temperature, low pressure
(b) High temperature, low pressure
(c) Low temperature, high pressure
(d) High temperature, high pressure
2. Rahul took a 100 ml beaker and filled half the beaker with 1
water and marked the level of water. He dissolved some salt
with the help of a glass rod and recorded the water level
again. Choose the correct observation related to above
activity:
(a) The water level increases appreciably
(b) The water level decreases
(c)The water level remains the same
(d)There is little increase in water level

3. Disha is asked to make a mixture with uniform composition. She is 1


provided with the following substances.
(i) water (ii) soil (iii) milk (iv) chocolate powder (v) sugar.
Which two substances should Disha mix to form a mixture with
uniform composition?
(a) (i) and (ii)
(b) (ii) and (iv)
(c) (ii) and (v)
(d) (i) and (v)

4. Study the diagrams representing mixtures, elements and compounds 1


shown, and choose the correct statement from the following:

(a) A represents an element and D represents a mixture.


(b) B represents a compound and C represents a mixture.
(c) B represents an element and D represents a mixture.
(d)A represents an element and C represents a mixture.

5. 1
ELEMENT A B C D
Mass 56 39 24 40
Number
Atomic 26 19 12 18
number
Which of the following correctly represents the number of electrons,
protons and neutrons in A, B, C and D respectively?
(i) Element A-26 electrons, 26 protons, 30 neutrons
(ii) Element B- 19 electrons, 19 protons, 19 neutrons
(iii) Element C-12 electrons, 12 protons, 24 neutrons
(iv) Element D- 18 electrons, 18 protons, 22 neutrons

(a) (i) and (ii)


(b) (ii) and (iii)
(c) (i) and (iv)
(d) (iii) and (iv)

6. Tritium, Deuterium, and protium are isotopes of 1

hydrogen. They have


(a) Similar physical properties.
(b) Similar chemical properties.
(c ) Both physical and chemical properties are similar.
(d) Both physical and chemical properties are different.

7. Identify the Mg2+ ion from the Figure where n and p represent the number of 1
neutrons and protons respectively

8. Engulfing of food materials or foreign bodies by cells like Amoeba is called: 1


(a) Diffusion
(b) Endocytosis
(c) Osmosis
(d) Plasmolysis
9. A de-shelled egg is placed in a concentrated salt solution and is 1
observed after five minutes. What change is likely to be
observed:
(a) The egg will swell
(b) The egg will shrink
(c) It will remain unchanged
(d) It will get plasmolysed

10. Actively dividing cells are present in: 1


(a) Xylem
(b)Phloem
(c) Cambium
(d) Collenchyma

11. If the tip of sugarcane plant is removed from the field, even then 1
it keeps on growing in length. It is due to the presence of:
a. Cambium
b. Apical meristem
c. Lateral meristem
d. Intercalary meristem

12. The living component present in permanent complex tissue 1


is/are:
a. Parenchyma and fibres
b. Only parenchyma
c. Tracheids and sieve tubes
d. Tracheids and companion cells

13. A car travels 10m in 5 seconds, 20m in next 10 seconds and 30n 1
in the last 10 seconds. The average speed of the motion is
(a) 2.0m/s
(b) 2.2m/s
(c) 2.2m/s
(d) 2.4m/s

14. In a tug of war, the work done by the winning team and the 1
losing team are respectively
(a) positive, positive
(b) positive, negative
(c) negative, positive
(d) zero, positive
15. The organelle that helps in detoxification of many drugs in liver 1
cells in vertebrates is:
a. SER
b. RER
c. ER
d. Lysosomes

16. Find out the correct statement: 1


i. Introducing genes of desired character into a plant gives
genetically modified crop.
ii. Cross between plants of two species is called inter-
varietal hybridization.
iii. Hybridization means crossing between genetically
dissimilar plants.
iv. Cross between two varieties is called inter-specific
hybridization.
(a) I and III
(b) II and IV
(c) I and IV
(d) I and II

Question No. 17 to 20 consist of two statements –


Assertion (A) and Reason (R). Answer
these questions selecting the appropriate option given
below:
a) Both A and R are true, and R is the correct
explanation of A.
b) Both A and R are true, and R is not the correct
explanation of A.
c) A is true but R is false.
d) A is false but R is true.

17. Assertion(A): Dust particles in air form aerosol. 1


Reason(R): Dust particles form dispersion medium and air is
dispersed phase.
Assertion: Plant tissues are mostly dead.
18. 1
Reason: Dead tissues can provide mechanical strength to the
plant.

19. Assertion(A): The ceiling of concert halls and conference halls 1


are made curved.
Reason(R): Sound after reflection from curved ceiling reaches
all parts of the hall.
20. Assertion: Tallness and profuse branching are desirable 1
characters for cereal crops.
Reason: Tallness helps in increasing the production of cereals.

Section B
Question No. 21 to 26 are very short answer questions

21. Calculate the amount of water required to prepare 500 g of 2


2.5% solution of sugar. Also write the formula used.

22. Explain what happens when a drop of concentrated sugar 2


solution is placed on a Rhoeo eaf peel mounted on a glass slide.
Name this phenomenon. Would the same happen if the Rhoeo
leaf was boiled before mounting?

23. Name one Indian and one exotic breed of honey bee that is used 2
for commercial honey production in India.
OR
Mention any two desirable characters of bee varieties suitable
for commercial honey production.

24. Give reason for the following: 2


(a) Passengers of a car are required to wear seat belt.
(b) As the sailor jumps in forward direction, the boat moves
backward.

25. A solid body of mass 8× 103 kg and volume 4 m3 is put in 2


water. Will the body float or sink. Justify your answer.

26. (a) Draw a wave pattern for high pitch and low pitch sound. 2
(b) An echo is heard in 3 seconds after the emission of sound in
air is 342 m/s, what is the distance of the reflecting surface from
the source?

Section C
Question No. 27 to 33 are short answer questions

27. Give reasons for the following: 3


(a) Camphor disappears if kept in open air for a few days.
( b) Outlet pipes of air conditioners drain more water on
humid days.
(c ) Water is kept in earthern pots during summers.
28. (a) Metal ‘X’ forms a phosphate with formula X3(PO4)2. What 3
will be the formula of the carbonate and hydroxide of metal ‘X’
(b) Give one example each for:
(i) Heteroatomic triatomic molecule
(ii) Triatomic molecule of element
OR
(a) Write the names of the following compounds:
(i) (NH4)2SO4 (ii)AlN
(b) Nitrogen and hydrogen combine in the ratio of 14:3 by
mass to form ammonia. What mass of nitrogen gas would be
required to react completely with 10.5 g of hydrogen gas?

29. Give three points of difference between a bacterial cell and a 3


typical plant cell.

30. . Given below is a diagram of a type of connective tissue present 3


in the trachea of human beings. Give the correct labeling of its
components X and Y. Specify the type of matrix it has.

31. (a) Establish the mathematical formula for gravitational 3


potential energy of a body of mass ‘m’ kept at ‘h’ distance
above the ground?
(b) What happens to the kinetic energy if the speed of the body
is halved?

32. (a) Find the pressure when a thrust of 20N is applied on surface 3
area of 20 cm2.
(b) An object is thrown up with a velocity of 20 m/s. Find the
maximum height attained by it and the total time taken by it to
come back.
33. Speed of sound travelling through three different media at a 3
particular temperature is given below observe the table and
answer the following questions

a) Is it possible to hear sound of an approaching train in


the rails but not in the air, much before the train comes.
Why?
b) Why is a sound wave called as longitudinal wave?
c) We receive heat and light from the Sun but cannot
hear sound of explosions occurring on its surface.
Give reason.

Section-D
Question No. 34 to 36 are long answer questions

34. (a) Observe the diagram carefully. 5


A student takes 6.2ml solution of ‘X’ in a small test tube which is
hung in a sealed conical flask, containing 5.8ml solution of ‘Y’.
The flask is slightly tilted so that the two solutions get mixed with
each other and a chemical reaction occurs.

(a) Which law of chemical combination is applicable here? State the


law.
(b) Calculate the ratio by number of atoms for sulphur
trioxide molecule. Given the ratio by mass of sulphur and
oxygen as 2:3.
(c) Calculate the formula unit mass of Al2(SO4)3.
[Atomic mass of Na=23u, H=1u, C=12u, O=16u)
OR
(a) Give the names of the elements present in the following
compounds.
(i) Quick lime
(ii) Baking soda
(b) Given below are the names and symbols of some species:

NAME OF SPECIES SYMBOL


Ammonium ion
Sodium ion
Carbon monoxide CO
Nitrate ion
Sulphite ion
Calcium ion

Oxide ion

Identify the following from species given in the table:


i) Monopositive polyatomic ion
ii) Atomicity of Nitrate ion
iii) Species with atomicity 2
iv) Ion formed by the loss of 2 electrons
(c ) Calculate the molecular mass of glucose(C6H12O6). Given the
atomic mass of C: 12u,H=1u,O=16 u
35. Sumit while playing football with his friends got Injured as his ankle 5
got twisted. His father took him to a doctor. After the initial
examination, the doctor told them that he was suffering from a sprain
due to tearing of ligament.
(a) What type of animal tissue a ligament is?
(b) State the difference between a ligament and a tendon on the basis
of their function and and flexibility.
(c) Name the type of muscular tissue present in his legs helped him
in playing. Give any two structural features of this type of muscle.

36. (a) Two workers do the same work. One completes the work 5
in 3 hours but the other takes 5 hours to complete the work.
Whose power is more and why.
(b) A bullet of mass 5 g travels with a speed of 500 m/s. If it
penetrates a fixed target which offers a constant resistance
force of 1000 N to the bullet. Find:
(i) the initial kinetic energy of the bullet.
(ii) the distance through which the bullet has penetrated.
OR
(a)State the law of conservation of energy and explain it for a
body of mass m falling from a point A from the height h,
above the ground.
(b)The masses of a scooter and a bike are in the ratio of 4:7.
Both are moving with different speed in the ratio 2:3.
Compute the ratio of their kinetic energy.

Section-E
Question No. 37 to 39 are case-based/data -based questions
with 2 to 3 short sub-parts.
Internal choice is provided in one of these sub-parts.

37. Meena’s research project on Thomson, Rutherford and Bohr 4


atomic models involved studying the key features, postulates
and significance of each model

Atomic Model Key features

Thomson ‘s Model J. J. Thomson, in 1898,


proposed that an atom
possesses a spherical
shape (radius
approximately 10–10 m) in
which the positive charge
is uniformly distributed. The
electrons are embedded into
it in such a manner as to give
the most stable electrostatic
arrangement

Rutherford ‘s Model Rutherford and his students


bombarded very thin gold
foil with α–particles.The
results of the scattering
experiment were quite
unexpected.
Electrons orbit the nucleus
in a planetary motion.

Bohr’s Model Neil Bohr put forward the


main postulates about the
model of an atom.
Electrons orbit the nucleus
in fixed energy shells K, L,
M, N etc

(a) Which part of Bohr's model of atomic structure explains


why electrons do not crash into the nucleus?
(b) What are α–particles?
(c) State the conclusions in Alpha particle scattering
experiment which were concluded from following
observations:
(i) Most of the α–particles passed straight through the gold
foil.
(ii) One out of 12000 particles rebound.
OR
(c) (i) Draw the electron distribution diagram of Carbon atom
according to Bohr’s model.
(ii) How can we say that atom is electrically neutral
according to Thomson’s model of an atom.
38. Rahul started driving his car. He increased the speed till 4 4
seconds and then he kept his car in constant speed for 6
seconds. Then after he decreased the speed of the car upto
another 6 seconds. After reaching at the starting place, he
draws the speed-time graph of his 16 seconds driving as shown
below:

a) What type of motion is represented by OA, AB


and BC?
b) What is the retardation of the motion of the
Rahul?
c) Calculate the distance travelled by him from 4 to
16seconds.
OR
A train running at 108 km/h is brought to a halt in 2
minutes. Calculate the distance the train travels before
stopping?

39. Swarn was growing wheat crop continuously year after year in 4
the same field. He observed that his crops were getting infected
by insects and yield becoming low year after year. He discussed
this problem with his father who advised him to grow crops in
pre-planned succession and grow legumes to increase the soil
fertility.
a. What was the reason behind the decrease in the yield of
the wheat crop .
b. How do leguminous plants increase the soil fertility?
c. Name any one legume crop.
d. What kind of cropping patterns (any two) suggested by
his father, he can opt to increase the overall yield?
SOCIAL SCIENCE
CLASS IX
SYLLABUS – FINAL
TERM 2023-24

S. SUBJECT NAME OF THE CHAPTERS


NO
I. HISTORY Ch-1.The French Revolution
Ch-2. Socialism in Europe and the Russian Revolution.
Ch-3. Nazism and the rise of Hitler
II. GEOGRAPHY Ch -1. India: Size and location.
Ch -2. Physical Features of India.
Ch-3. Drainage
Ch -4. Climate.
Ch- 6. Population.

III. POLITICAL Ch--1. What is Democracy? Why Democracy?


SCIENCE Ch- 2. Constitutional Design
Ch - 3. Electoral Politics
Ch - 4. Working of the Institutions
Ch- 5. Democratic Rights
IV. ECONOMICS Ch - 2 People as Resource
Ch – 3. Poverty as a Challenge.
Ch – 4. Food Security in India
SOCIAL SCIENCE
SAMPLE PAPER
ANNUAL EXAMINATION
BLUE PRINT (2023-24)
CLASS- IX

HISTORY - 20 MARKS

GEOGRAPHY- 20 MARKS

CIVICS - 20 MARKS

ECONOMICS-20 MARKS

TOTAL - 80 MARKS

VSA VSA SA LA SOURCE MAP TOTAL


1 MARK 2 3 5 BASED QUESTIONS
MARKS MARKS MARKS QUESTIONS
HISTORY 2 Marks
CH-1 THE FRENCH 2(1) 1(4)
REVOLUTION

CH-2 SOCIALISM 1(1) 1(2) 1(3)


IN EUROPE AND
THE RUSSIAN
REVOLUTION

CH-3 NAZISM 1(1) 1(5)


AND THE RISE OF
HITLER

4(1) 1(2) 1(3) 1(5) 1(4) 2(1) 20


GEOGRAPHY 3 Marks
CH-1 INDIA: SIZE 2(1)
AND LOCATION

CH-2 PHYSICAL 1(1) 1(2)


FEATURES OF
INDIA
CH-3 DRAINAGE 1(3)

CH-4 CLIMATE 1(5)

CH-6 1(4)
POPULATION
3(1) 1(2) 1(3) 1(5) 1(4) 3(1) 20
CIVICS
CH-1 WHAT IS 4(1)
DEMOCRACY?
WHY
DEMOCRACY?
CH-2 2(1) 1(2)
CONSTITUTIONAL
DESIGN
CH-3 ELECTORAL 1(3)
POLITICS
CH-4 WORKING 2(1) 1(2)
OF THE
INSTITUTION
CH-5 1(5)
DEMOCRATIC
RIGHTS
8(1) 2(2) 1(3) 1(5) 20
ECONOMICS
CH-2 PEOPLE AS 2(1) 1(5)
A RESOURCE
CH-3 POVERTY AS 3(1) 1(3)
A CHALLENGE
CH-4 FOOD 1(3) 1(4)
SECURITY IN
INDIA
5(1) 2(3) 1(5) 1(4) 20

NOTE- MAP TEST ITEMS


NO MAP FROM CH- NATURAL VEGETATION AND WILDLIFE

IMPORTANT NOTE *CH- CLIMATE : HIGHEST & LOWEST RAINFALL AREAS

DIRECTIONS OF WINDS

CH- POPULATION : HIGHEST & LOWEST DENSITY OF POPULATION IN INDIA

REST AS PER CBSE LIST.


CLASS IX – SESSION 2023-24
SOCIAL SCIENCE SAMPLE PAPER (CODE 087)
Maximum Marks: 80
General Instructions:
1. The question paper comprises Six Sections – A, B, C, D, E and F. There are 37 questions
in the Question paper. All questions are compulsory.
2. Section A – From questions 1 to 20 are MCQs of 1 mark each.
3. Section B – Question no. 21 to 24 are Very Short Answer Type Questions, carrying 2
marks each. Answers to each question should not exceed 40 words.
4. Section C contains Q.25to Q.29 are Short Answer Type Questions, carrying 3 marks
each. Answer to each question should not exceed 60 words
5. Section D – Question no. 30 to 33 are long answer type questions, carrying 5 marks each.
Answers to each question should not exceed 120 words.
6. Section-E - Questions no from 34 to 36 are case based questions with three sub
questions and are of 4 marks each. Answers to each question should not exceed 100 words.
7. Section F – Question no. 37 is map based, carrying 5 marks with two parts, 37a from
History (2 marks) and 37b from Geography (3 marks).
8. There is no overall choice in the question paper. However, an internal choice has been
provided in few questions. Only one of the choices in such questions have to be attempted.
9. In addition to this, separate instructions are given with each section and question,
wherever necessary.

Section A

1 In the context of France the volunteers from Marseilles sang the 1


Marseillaise, a patriotic song when they marched into Paris. Who
composed this song?
a) Maximilian Robespierre
b) Marie Antoinette
c) Roget de L’Isle
d) Mirabeau

2 Find the odd one out 1


a) Terai
b) Khader
c) Doab
d) Bhangar.

3 Which statement is not correct in relation to seasonal unemployment 1


a) When people are not able to find jobs during some months of the
year
b) People dependent on agriculture usually face such kind of problem
c) People appear to be employed
d) They are only busy at the time of sowing and harvesting etc

4 Which group of states is correct in reference to passing of Tropic of 1


Cancer.
a)Gujarat,Madhya Pradesh, Chhattisgarh, Orissa, West Bengal
b)Gujarat, Rajasthan, Madhya Pradesh,Jharkhand, Chhattisgarh
c)Gujarat,Madhya Pradesh,Manipur, Orissa, West Bengal
d)Gujarat,Madhya Pradesh, Chhattisgarh,Bihar,West Bengal

5 1
Match the columns:
COLUMN A COLUMN B

1. Cabinet ministers a) Lok Sabha

2. Council of States b) Permanent executive

3. Civil servants c) Political executive

4. House of the people d) Rajya sabha


OPTIONS:
a)1-d, 2-c, 3-a, 4-b c) 1-b, 2-a, 3-c, 4-d
b)1-c, 2-d, 3-a, 4-b d) 1-c, 2-d, 3-b, 4-a

6 Estonia has made its citizenship rules in such a way that people 1
belonging to Russian minority find it difficult to get the
a. Economic Rights
b. Right to follow their Religion
c. Right to Vote
d. Right to Education

7 In the question given below, there are two Statements marked as 1


Assertion (A) and Reason (R). Read the Statements and Choose the
correct option: Options are:
(a) Both (A) and (R) are true and (R) is the correct explanation of (A).
(b) Both (A) and (R) are true but (R) is not the correct explanation of (A).
(c) (A) is correct but (R) is wrong.
(d) (A) is wrong but (R) is correct.
Assertion: The Indian Constitution was heavily influenced by the US
Constitution.
Reason: The framers of the Indian Constitution were inspired by the
ideas of liberty, equality, and democracy enshrined in the US
Constitution.

8 Which of the following is not a Anti-poverty programme 1


a) MNREGA
b) AAY
c) NSSO
d) PMGY

9 The head of the government in Nepal is the: 1


a) President
b) Prime Minister
c) King
d) Vice President
10 Which of the following aspects best signifies this image? 1

a. Political freedom for workers


b. Improved living conditions of the workers
c. Equality for women
d. The car Volkswagen

11 Why is literacy rate low in females? 1


a) Lack of equal education opportunities
b) Lack of transport facilities
c) Lack of infrastructure
d) Lack of income

12 Which of these positions is correct in relation to the ‘Sovereign’ 1


status of India?
(a) USA can decide India’s foreign policy
(b) USSR can support the CPI (M) in setting up its government here
(c) The Indian government only can decide its internal and external
policies
(d) Pakistan can control India’s Armed Forces

13 Arrange the following in correct sequence 1


1. Return of Lenin
2. October revolution
3. Russia’s peace with Germany
4. Establishment of Provisional Government

a) 4-2-1-3
b) 1-3-2-4
c) 2-4-3-1
d) 4-1-2-3

14 The calorie requirement is higher in the rural areas because 1


(a)They do not enjoy as much as people in the urban areas.
(b) Food items are expensive.
(c) They are engaged in mental work.
(d) People are engaged in physical work.
15 Who led the Bolshevik group in Russia during the Russian 1
Revolution?

(a) Karl Marx

(b) Friedrich Engels

(c) Vladimir Lenin

(d) Trotsky

16 Identify the state which does not have international boundary- 1


a) Haryana
b) Uttarakhand
c) Sikkim
d) Bihar

17 Which of the following sentences is correct? 1


(a) All countries that have constitutions are necessarily democratic
(b) All countries that are democratic necessarily have constitutions
(c) Both (a) and (b)
(d) None of the above)

18 Apartheid in South Africa was discrimination on the basis of 1


(a) Gender
(b) Religion
(c) Race
(d) Economic status
19 The strength of the Council of Ministers ranges from 1
(a) 60 to 80
(b) 60 to 100
(c) 70 to 85
(d) 80 to 100

20 1

Study the graph given below and answer the following question

Identify the state with Highest poverty ratio-

a) Odisha
b) Assam
c) Bihar
d) Madhya Pradesh

Section B

VERY SHORT ANSWER QUESTION (2 × 4= 8)

21 Explain any two features of the Preamble of India. 2

22 Describe the Political conditions in Russia before 1905. 2

23 Compare any two features of the Coastal Plains of India. 2

24 Write any two reactions of the people on the recommendations of the 2


Mandal Commission.

Section C

SHORT ANSWER-BASED QUESTIONS (3 x 5= 15)

25 Differentiate between the ideas of Liberals and Radicals in Europe. 3


26 Analyse any three measures used by different states to reduce poverty in 3
India.

27 ‘Indus is a major river of Himalayan River System’. Mention its - 3


-Origin point
- Tributaries
-State in which its basin is located

28 Give details of a legal declaration given by a candidate who is contesting 3


elections.

29 Mention the three dimensions of Food security. 3

Section D

LONG ANSWER-BASED QUESTIONS (5 x 4= 20)

30 Compare the characteristic features of Advancing and Retreating 5


Monsoon.

31 ‘In my state the mother is the most important citizen.’ Explain what role 5
women had in Nazi society.

OR
‘Hitler was fanatically interested in the youth of the country. He felt that a
strong Nazi society could be established only by teaching children Nazi
ideology.” Illustrate the statement.

32 Describe citizen’s rights in Saudi Arabia. 5


OR
State the importance of Rights in Democracy.

33 Elaborate some of the measures taken by the Indian government to 5


improve the Education system in India.
OR
In what ways Unemployment leads to wastage of manpower.

Section E

CASE-BASED QUESTIONS ( 4 x 3= 12)

34 The period from 1793 to 1794 is referred to as the Reign of Terror. pg16
Robespierre followed a policy of severe control and punishment. All those
whom he saw as being ‘enemies’ of the republic – ex-nobles and clergy,
members of other political parties, even members of his
own party who did not agree with his methods – were arrested,
imprisoned and then tried by a revolutionary tribunal. If the court found
them ‘guilty’ they were guillotined.
Robespierreís government issued laws placing a maximum ceiling on
wages and prices. Meat and bread were rationed. Peasants were forced
to transport their grain to the cities and sell it at prices fixed by the
government. The use of more expensive white flour was forbidden; all
citizens were required to eat the pain díÈgalitÈ (equality bread), a loaf
made of wholewheat. Equality was also sought to be practised through
forms of speech and address. Instead of the traditional Monsieur (Sir)
and Madame (Madam) all French men and women were henceforth
Citoyen and Citoyenne (Citizen).
34.1 Why is the period from 1793 to 1794 termed as the Reign of Terror? 1
34.2 What is a guillotine? 1
34.3 Mention any two laws implemented by Robespierre to bring equality and 2
liberty in the French Society.

35 Adolescents are the most important resource for the future. Nutrition
requirements of adolescents are higher than those of a normal child or
adult. Poor nutrition can lead to deficiency and stunted growth.
The adolescent girls have to be sensitised to the problems they confront.
Their awareness can be improved through the spread of literacy and
education among them.
The NPP 2000 provides a policy framework for imparting free and
compulsory school education up to 14 years of age, reducing infant
mortality rate to below 30 per 1000 live births, achieving universal
immunisation of children against all vaccine preventable diseases,
promoting delayed marriage for girls, and making family welfare a
people-centred programme. 1
35.1 Which age group is a part of the adolescence age group. 1
35.2 Mention any one objective of NPP 2000. 2
35.3 How can girls be sensitised towards the problems they confront?
36 The PDS has proved to be the most effective instrument of government
policy over the years in stabilising prices and making food available to
consumers at affordable prices. It has been instrumental in averting
widespread hunger and famine by supplying food from surplus regions of
the country to the deficit ones. In addition, the prices have been under
revision in favour of poor households in general. The system, including
the minimum support price and procurement has contributed to an
increase in food grain production and provided income security to
farmers in certain regions.

36.1 How does MSP benefit farmers? 1


36.2 Explain the benefits of PDS. 1
36.3 2
State the grounds on which the PDS system has faced severe criticism.

Section F

MAP SKILL-BASED QUESTION (2+3=5)

37 I. On a given outline map of France name and locate the following: 1+1
A.1 A) The Fortress Prison Bastille was located in this city.

II. On a given outline map of the World name and locate the following:
B) This country was a part of Allied Powers during World War I.

Map for 37.1(I)


Map for 37.1 (II)
37.2 On the given political map of India, locate and label ANY THREE 3
locations out of the following:

1) Malabar coast
2) Pulicat Lake
3) Area with the lowest rainfall
4) State with the highest density of population
CLASS IX – SESSION 2023-24
SOCIAL SCIENCE ANSWER KEY (CODE 087)
Maximum Marks: 80

Section A

1 (c) Roget de L’Isle 1

2 (c) Doab 1

3 (c) People appear to be employed 1

4 (b) Gujarat, Rajasthan, Madhya Pradesh,Jharkhand, Chhattisgarh 1

5 (b) 1-c, 2-d, 3-a, 4-b 1

6 (c) Right to Vote 1

7 (a) Both (A) and (R) are true and (R) is the correct explanation of (A). 1

8 (c) NSSO 1

9 (a) President 1

10 (b) Improved living conditions of the workers 1

11 (a) Lack of equal education opportunities 1

12 (c) The Indian government only can decide its internal and external 1
policies

13 (a) 4-2-1-3 1

14 (d) People are engaged in physical work. 1

15 (c) Vladimir Lenin 1

16 (a)Haryana 1

17 (b)All countries that are democratic necessarily have constitutions 1

18 (c) Race 1

19 (a) 60 to 80 1

20 (a) Odisha 1
21 SOVEREIGN People have supreme right to make decisions on internal 2
as well as external matters. No external power can dictate the
government of India.
SOCIALIST Wealth is generated socially and should be shared equally
by society. Government should regulate the ownership of land and
industry to reduce socio-economic inequalities.
SECULAR Citizens have complete freedom to follow any religion. But
there is no official religion. Government treats all religious beliefs and
practices with equal respect
DEMOCRATIC A form of government where people enjoy equal political
rights, elect their rulers and hold them accountable. The government is
run according to some basic rules.
REPUBLIC The head of the state is an elected person and not a
hereditary position.
JUSTICE Citizens cannot be discriminated on the grounds of caste,
religion and gender. Social inequalities have to be reduced. Government
should work for the welfare of all, especially of the disadvantaged
groups.
LIBERTY There are no unreasonable restrictions on the citizens in what
they think, how they wish to express their thoughts and the way they
wish to follow up their thoughts in action.
EQUALITY All are equal before the law. The traditional social
inequalities have to be ended. The government should ensure equal
opportunity for all.
FRATERNITY All of us should behave as if we are members of the same
family. No one should treat a fellow citizen as inferior
(any two with explanations)

22 -Industry was found in pockets. Prominent industrial areas were St Pr


Petersburg and Moscow. Craftsmen undertook much of the production, 30
but large factories existed alongside craft workshops. Many factories 2
were set up in the 1890s, when Russia’s railway network was extended,
and foreign investment in industry increased. Coal production doubled
and iron and steel output quadrupled. By the 1900s, in some areas
factory workers and craftsmen were almost equal in number.

- Most industry was the private property of industrialists. Government


supervised large factories to ensure minimum wages and limited hours of
work. But factory inspectors could not prevent rules being broken. In craft
units and small workshops, the working day was sometimes 15 hours,
compared with 10 or 12 hours in factories. Accommodation varied from
rooms to dormitories.

23 Coastal Plain: 2
-Western coast plain lies between the western ghat and Arabian sea.
Eastern coast plain lies between eastern ghats and Bay of Bengal
-Eastern Coast is sandwiched between the Eastern Ghats and Bay of
Bengal
- The northern part of Western Coast is called Konkan and Southern is
called Malabar Coast.
-The Eastern Coast is referred as Northern Circars
( Any two points)

24 - It led to widespread protests and counter protests, some of which were 2


violent.

-People reacted strongly because this decision affected thousands of job


opportunities.

-Some felt that the existence of inequalities among people of different


castes in India necessitated job reservations. They felt, this would give a
fair opportunity to those communities who so far had not adequately
been represented in government employment.

-Others felt that this was unfair as it would deny equality of opportunity
to those who did not belong to backward communities.

-Some felt that this would perpetuate caste feelings among people and
hamper national unity.

(Any two points)

25 Liberals 3
-Liberals wanted a nation which tolerated all religions.
-Liberals also opposed the uncontrolled power of dynastic rulers
-They wanted to safeguard the rights of individuals against governments
-They argued for a representative, elected parliamentary government,
subject to laws interpreted by a well-trained judiciary that was
independent of rulers and officials
- They did not believe in universal adult franchise, that is, the right of
every citizen to vote.
Radicals
-Radicals wanted a nation in which the government was based on the
majority of a country’s population.
-Many supported women’s suffragette movements.
-They opposed the privileges of great landowners and wealthy factory
owners.
-They were not against the existence of private property but disliked
concentration of property in the hands of a few.

26 Measures to reduce poverty in Kerala, Maharashtra, Andhra Pradesh, Pg


Tamil Nadu, Gujarat and West Bengal. States like Punjab and Haryana 35
have traditionally succeeded in reducing poverty with the help of high 3
agricultural growth rates. Kerala has focused more on human resource
development. In West Bengal, land reform measures have helped in
reducing poverty. In Andhra Pradesh and Tamil Nadu public distribution
of food grains could have been responsible for the improvement.

27 INDUS RIVER Pg
Origin- Rises in Tibet near lake Mansarowar 18
Tributaries- Zaskar, Nubra, Satluj etc.. (any two) 3
River basin- HImachal Pradesh and Punjab

28 Every candidate has to make a legal declaration, giving full details of : Pg


- Serious criminal cases pending against the candidate 42
- Details of the assets and liabilities of the candidate and his or her family 3
- Educational qualifications of the candidate.

29 -Availability of food means food production within the country, food


imports and the previous year's stock stored in government granaries.

-Accessibility means food is within reach of every person.

-Affordability implies that an individual has enough money to buy


sufficient, safe and nutritious food to meet one's dietary needs.
1

Section D

LONG ANSWER-BASED QUESTIONS (5 x 4= 20)

30 Advancing Monsoon Pg
1. Starts in June and covers the whole country by July. 32
2. It has two branches South West and South East.
3. The wind moves from sea to land and is moisture laden.
4. It brings heavy to light rainfall throughout the whole country.
5. It has a unique feature of break in or dry spell.
Retreating Monsoon
1. It starts in September end and ends in November/December.
2. It has one branch North West to South East
3. The wind moves from land to sea and are dry winds
4. It brings rainfall to coastal region on Tamil Nadu and is associated
with Cyclones
5. It has a unique feature known as October heat which is
oppressive heat generated in this season.
(Any other relevant point)

31 In Nazi Germany all mothers were not treated equally. Pg


-Women who bore racially undesirable children were punished and those 68
who produced racially desirable children were awarded.
-They were given favoured treatment in hospitals and were also entitled
to concessions in shops and on theatre tickets and railway fares.
-To encourage women to produce many children, Honour Crosses were
awarded. A bronze cross was given for four children, silver for six and
gold for eight or more
-All Aryan women who deviated from the prescribed code of conduct
were publicly condemned, and severely punished. Those who
maintained contact with Jews, Poles and Russians were paraded
through the town with shaved heads, blackened faces and placards
hanging around their necks announcing ‘I have sullied the honour of the
nation’.
-Many received jail sentences and lost civic honour as well as their
husbands and families for this ‘criminal offence’
(any other relevant point)
OR
Hitler was fanatically interested in the youth of the country. Pg
66
- He felt that a strong Nazi society could be established only by teaching
children Nazi ideology.
-All schools were ëcleansedí and purified. This meant that teachers who
were Jews or seen as politically unreliableí were dismissed. Children
were first segregated: Germans and Jews could not sit together or play
together.
-Subsequently, undesirable children-Jews, the physically handicapped,
Gypsies were thrown out of schools. And finally in the 1940s, they were
taken to the gas chambers.
- School textbooks were rewritten. Racial science was introduced to
justify Nazi ideas of race.
- Even the function of sports was to nurture a spirit of violence and
aggression among children. Hitler believed that boxing could make
children iron hearted, strong and masculine
(Any other relevant point)

32 -The country is ruled by a hereditary king and the people have no role in pg
electing or changing their rulers. 98
- The king selects the legislature as well as the executive. He appoints 5
the judges and can change any of their decisions.
- Citizens cannot form political parties or any political organisations.
Media cannot report anything that the monarch does not like.
- There is no freedom of religion. Every citizen is required to be Muslim.
Non-Muslim residents can follow their religion in private, but not in public.
- Women are subjected to many public restrictions. The testimony of one
man is considered equal to that of two women.
OR
-Rights are necessary for the very sustenance of a democracy. In a
democracy every citizen has to have the right to vote and the right to be Pg
elected to government. 100
-For democratic elections to take place, it is necessary that citizens
should have the right to express their opinion, form political parties and
take part in political activities
-Rights also perform a very special role in a democracy. Rights protect
minorities from the oppression of majority.
-They ensure that the majority cannot do whatever it likes. Rights are
guarantees which can be used when things go wrong.
- Things may go wrong when some citizens may wish to take away the
rights of others. This usually happens when those in majority want to
dominate those in minority.
- The government should protect the citizens’ rights in such a situation.

(any other relevant point)

33 -There is a provision made for providing universal access, retention and Pg


quality in elementary education with a special emphasis on girls 21&2
-There is also an establishment of pace setting of schools like Navodaya 2
Vidyalaya in each district.
- Vocational streams have been developed to equip a large number of
high school students with occupations related to knowledge and skills.
-“Sarva Siksha Abhiyan is a significant step towards providing
elementary education to all children in the age group of 6–14 years
-Mid-day meal scheme has been implemented to encourage attendance
and retention of children and improve their nutritional status. These
policies could add to the literate population of India
(any other relevant point)
OR
-Unemployment leads to wastage of manpower resource. People who
are an asset for the economy turn into a liability. pg25
-There is a feeling of hopelessness and despair among the youth. People
do not have enough money to support their family
-Inability of educated people who are willing to work to find gainful
employment implies a great social waste.
-The dependence of the unemployed on the working population
increases.
- The quality of life of an individual as well as of society is adversely
affected.
(any other relevant point)

Section E

CASE-BASED QUESTIONS ( 4 x 3= 12)


34 pg16
34.1 The period from 1793 to 1794 is referred to as the Reign of Terror.
Robespierre followed a policy of severe control and punishment. 1

34.2 The guillotine is a device consisting of two poles and a blade with which
a person is beheaded. It was named after Dr Guillotin who invented it. 1

34.3 -Robespierreís government issued laws placing a maximum ceiling on 2


wages and prices. Meat and bread were rationed. Peasants were forced
to transport their grain to the cities and sell it at prices fixed by the
government.
-The use of more expensive white flour was forbidden; all citizens were
required to eat the pain díÈgalitÈ (equality bread), a loaf made of
wholewheat.
(any other relevant point)

35 Pg
no
35.1 Adolescents are generally grouped in the age-group of 10 to 19 years. 59
The NPP 2000 provides a policy framework for imparting free and
35.2
compulsory school education up to 14 years of age, reducing infant
mortality rate to below 30 per 1000 live births, achieving universal
immunisation of children against all vaccine preventable diseases, Pg
promoting delayed marriage for girls, and making family welfare a 53
people-centred programme.

35.3 -The adolescent girls have to be sensitised to the problems they


confront. -Their awareness can be improved through the spread of Pg
literacy and education among them. 59

2
36 Pg
50
36.1 The minimum support price and procurement has contributed to an
increase in food grain production and provided income security to 1
farmers in certain regions
36.2
The PDS has proved to be the most effective instrument of government 1
policy over the years in stabilising prices and making food available to
consumers at affordable prices

36.3 The Public Distribution System has faced severe criticism on several 2
grounds. Instances of hunger are prevalent despite overflowing
granaries.

FCI go-downs are overflowing with grains, with some rotting away and
some being eaten by rats.

Section F

MAP SKILL-BASED QUESTION (2+3=5)

37.1 37.1 (I) 1+1


A- PARIS
37.1 (II)
B- ENGLAND
3

Map answer for 37.1 (I)


Map answer for 37.1 (II)
Map answer for 37.2
कक्षा नवमी
संस्कृतम ् ( सम्प्रेषणात्मकम ् ) कोडसंख्या 119
प्रश्नपत्रस्य प्रारूपम ्
( 2023 - 2024 )

प्रश्नप्रकाराः प्रश्नानाम ् संख्या विभाग-संख्या अंक कुलांकाः


भारः

बहुविकल्पात्मकाः 1 अङ्कः 3+4+4+4+3+4=22 6 1 22


अति-लघत्त
ू रात्मकाः ½ अङ्कः 2+2+2=6 3 1/2 3
अति-लघत्त
ू रात्मकाः 1 अङ्कः 2=2 1 1 2
निबन्धात्मकाः ½ अङ्कः 10+10+4+8=32 4 1/2 16
( रिक्तस्थानपर्ति
ू माध्यमेन )
दीर्घोत्तरात्मकाः 1 अङ्कः 5+2+2+2+5=16 5 1 16
दीर्घोत्तरात्मकाः 2 अङ्कौ 2=2 1 2 16
लघत्त
ू रात्मकाः ½ अङ्कः 4+4+4=12 3 1/2 6
लघत्त
ू रात्मकाः 1 अङ्कः 2+2+2+1+4=11 5 1 11
80
आदर्श प्रश्नपत्रम ् ( 2023- 24)
कक्षा नवमी
संस्कृतम ् ( सम्प्रेषणात्मकम ् )
कोड सङ्ख्या - 119

समयः- होरात्रयम ् पर्णां


ू ङ्काः 80

सामान्यनिर्देशाः -
1. कृपया सम्यक्तया परीक्षणं कुर्वन्तु यत ् अस्मिन ् प्रश्नपत्रे 12 पष्ृ ठानि सन्ति।
2. कृपया सम्यक्तया परीक्षणं कुर्वन्तु यत ् अस्मिन ् प्रश्नपत्रे 19 प्रश्नाः सन्ति ।
3. अस्मिन ् प्रश्नपत्रे चत्वारः खण्डाः सन्ति।

क भागः अपठितावबोधनम ् 10 अङ्काः


ख भागः रचनात्मककार्यम ् 15 अङ्काः
ग भागः अनप्र
ु यक्
ु तव्याकरणम ् 25 अङ्काः
घ भागः पठितावबोधनम ् 30 अङ्काः

4 प्रत्येकं भागम ् अधिकृत्य उत्तराणि एकस्मिन ् स्थाने क्रमेण लेखनीयानि ।


5 उत्तरलेखनात ् पर्वं
ू प्रश्नस्य क्रमाङ्कः अवश्यं लेखनीयः ।
6 प्रश्नस्य क्रमांकः प्रश्नपत्रानस ु ारम ् एव लेखनीयः ।
7 सर्वेषां प्रश्नानाम ् उत्तराणि संस्कृतेन लेखनीयानि ।
8 प्रश्नानां निर्देशाः ध्यानेन अवश्यं पठनीयाः ।

क भागः
अपठितावबोधनम ् 10 अङ्काः
1 अधोलिखितं गद्यांशं पठित्वा प्रदत्तप्रश्नानाम ् उत्तराणि संस्कृतेन लिखत-
मानवः संसर्गशीलः भवति । सामाजिकः प्राणी अपि मानवसंसर्गेण विना जीवितंु न
शक्नोति । सज्जनाः दर्जु नाः वा सर्वे एव संगतिम ् इच्छन्ति । संसर्गस्य महती
आवश्यकता वैशिष्ट्यं चास्ति । संसर्गतः एव मनष्ु येषु गण ु ाः दोषाश्च समाविशन्ति । यदा
मनष्ु यः कुसंगतिं करोति तदा सः दर्जु नः भवति यदा च सत्संगतिम ् करोति तदा सः
सज्जनः भवति । संसर्गकारणात ् एव सज्जनाः दर्ज ु नाः भवन्ति दर्ज ु नाः च सज्जनाः
जायन्ते । कीटः अपि सम ु नसा सह सतां शिरः आरोहति । अतः अस्माभिः सदा सत्संगतिः
करणीया ।
( अ) एकपदे न उत्तरत-( केवलं प्रश्नद्वयम ् ) 1x2=2
(i) कीटः केषां शिरः आरोहति ?
(ii) मानवः कीदृशः प्राणी अस्ति?
(iii) सदा का करणीया ?

( आ ) पर्ण
ू वाक्येन उत्तरत- (केवलं प्रश्नद्वयम ् ) 2x2=4
(i) सत्संसर्गस्य के लाभाः सन्ति ?
(ii) कः मानवसंसर्गेण विना जीवितंु न शक्नोति ?
(iii) सर्वे जनाः किम ् इच्छन्ति ?

( इ)अस्य अनच्
ु छदे स्य कृते उपर्युक्तं शीर्षकं संस्कृतेन लिखत। 1x1=1

(ई) निर्देशानस ु ारम ् उत्तरत - (केवलं प्रश्नत्रयम ् ) 1x3=3


(i) 'दर्ज
ु नानां ' इति पदस्य विलोमपदं किम ् अस्ति ?
(क) सतां (ख) सत्संगतिः
( ग) शिरः (घ) कीटः

(ii) मानवः संसर्गशीलः भवति ।अत्र 'भवति ' इति क्रियापदस्य कर्तृपदम ् किम ् ?
(क) संसर्गशीलः (ख) मानवः
(ग) भवति (घ) समु नसा

(iii) 'अस्माभिः सदा सत्संगतिः करणीया ।'अस्मिन ् वाक्ये किं अव्ययपदम ् लिखितम ् ?
(क) अस्माभिः (ख) सत्संगतिः
(ग) करणीया (घ) सदा

(iv) सामाजिकः इति विशेषणपदस्य विशेष्यपदं किम ् ?


(क) महती (ख) आवश्यकता
(ग) संसर्गतः (घ ) प्राणी

ख भागः
रचनात्मककार्यम ् 15 अङ्काः

2 रमेशस्य अनजु ः अतलु ः अम्बालानगरे छात्रावासे निवसति । रमेशन े समयस्य


सदपु योगाय अतलु ाय लिखितम ् इदं पत्रं मञ्जष
ू ायां दत्तैः पदै ः परू यित्वा उत्तरपस्ति
ु कायां
लिखत- 1/2x10=5
(i)_______
दिनांकः_______

प्रिय अतल
ु !
(ii) ________।

अत्र कुशलं तत्र (iii)_____ अतल ु ! मानवजीवनस्य प्रतिपलं (iv ) _____ अस्ति ।
विश्वे प्रकृतेः सम्पर्णा
ू नि कार्याणि नियमितसमये (v)_____। उचितमसये यदि वष्टि ृ ःन
भवेत ् तदा सर्वं क्षेत्रं शष्ु येत ् । तथैव बाल्यकाले यदि छात्रः न पठति तदा असौ
समस्तजीवने (vi)_____ करोति ।समयः तस्य एव जनस्य कार्येषु सहभागितां करोति यः
(vii)_____ सदप ु योगं करोति । गते काले शोकः (viii) _____। समयः कदापि
कस्यचिदपि (xi)_____ न करोति । अतः सदा समयस्य सदप ु योगः करणीयः ।

भवतः (x)_______
रमेशः
मञ्जष ू ा-
( निरर्थकः, अग्रजः, शभ ु ाशीषः, दिल्लीनगरतः, महत्त्वपर्णू म ्, समयस्य, पश्चात्तापम ्,
भवन्ति, प्रतीक्षाम ्, अस्तु )
3 प्रदत्तं चित्रं दृष्ट्वा मञ्जषू ायां प्रदत्तपदानां सहायतया पंच वाक्यानि संस्कृतेन लिखत-
1x5=5

मञ्जष
ू ा-
( गणतन्त्रदिवसः, जनाः, ध्वजः, भारतद्वारम ्, सर्वत्र, सैनिकाः, तिष्ठन्ति, वक्ष
ृ ाः,
पश्यन्ति, स्वच्छं , गगनम ्, उत्साहिताः )
अथवा
'स्वाधीनतायाः पीडाकथा' इति विषयम ् अधिकृत्य मञ्जष ू ाप्रदत्तपदानां सहायतया पंचभिः
संस्कृतवाक्यैः एकम ् अनच्
ु छे दं लिखत- 1x5=5

मञ्जष ू ा
(पंक्तौ , वर्षाणि, उपविशन्ति , नाम, स्वाधीनता, भ्रष्टाचारे ण, पीडिता, जनता, त्रस्ता,
अधिकारिणः, मन्त्रिणः, भ्रष्टाः, अहम ्, चलितम
ु ् , जनाः )

4 मञ्जष ू ायाः सहायतया अधोलिखितां कथां परू यित्वा पनु ः लिखत- 1/2x10=5
श्रीरामः (i)_____ आसीत ् । सः (ii) _____दशरथस्य पत्र ु ः आसीत ् । (iii) _____ आज्ञां
पालयितंु सः चतर्द ु शवर्षाणि यावत ् वने (iv)______ अकरोत ्, तेन सह तस्य (v)_____
सीता, भ्राता लक्ष्मणः चापि (vi)______ अगच्छताम ् । वने (vii) ____ सीताम ् अहरत ् ।
रामः (viii)______ हत्वा सीताम ् आनयत ् । सः (ix)______ विभीषणाय अयच्छत ् ।
रामायणे श्रीरामस्य (x)______वर्णिता अस्ति ।

मञ्जषू ा-
( रावणः, रावणम ्, जीवनकथा, मर्यादापरु ु षोत्तमः, पितःु , वनम ्, निवासम ्, राज्ञः,
लंकाराज्यम ्, पत्नी )
अथवा
रिक्तस्थानानि परू यित्वा अधोलिखितसंवादं पन ु ः लिखत- 1x5=5
अध्यापकः – हे दे वेश ! किं त्वं जानासि यत ् हिन्दन ू ां प्रमख
ु ः उत्सवः कः अस्ति
दे वेशः - आम ्,(i)__________________।
अध्यापकः - दे वेश! किमर्थं मान्यते अयम ् उत्सवः?
दे वेशः - (ii)_______________________।
अध्यापकः - त्वया उचितम ् उक्तम ्। अस्मिन ् दिने जनाः किं कुर्वन्ति?
दे वेशः - (iii)________________________।
अध्यापकः - (iv)_____________________।
दे वेशः - रात्रौ लक्ष्मीपज
ू नं भवति ।
अध्यापकः - यथा सरस्वती विद्यायाः दे वी, तथा लक्ष्मीः कस्य दे वी?
दे वेशः - गरु
ु वर ! (v)__________________।
अध्यापकः – शोभनम ्! अधन ु ा गह
ृ ं गच्छ ।

मञ्जष
ू ा-
(i) दीपावल्यां रात्रौ किं भवति?
(ii) लक्ष्मीः धनस्य दे वी अस्ति।
(iii) दीपावलिः हिन्दन ू ां प्रमख
ु ः उत्सवः अस्ति ।
(iv) अस्मिन ् दिने श्रीरामचन्द्रः अयोध्याम ् आगच्छत ् ।
(v) अस्मिन ् दिने जनाः स्वगह ृ ाणि भष
ू यन्ति । दीपकान ् प्रज्ज्वालयन्ति ।

ग भागः
अनप्र
ु यक्
ु तव्याकरणम ् 25 अङ्काः

5 निर्देशानस
ु ारम ् उत्तरत- (केवलं प्रश्नचतष्ु टयम ् ) 1/2x4=2
(i) आ, च ्, म ्, य ् वर्णेषु कः वर्णः कण्ठ्यः अस्ति ?

(ii) कमलम ् इति पदे म ् इति पदस्य उच्चारणस्थानं किम ् ?

(iii) प ् इति वर्णस्य उच्चारणस्थानं किम ् ?

(iv) स ् इति वर्णः दन्त्यः अस्ति अथवा मर्ध


ू न्यः ?

(v) इ, फ् , य ्, श ्, छ् वर्णेषु कः वर्णः तालव्यः न अस्ति ?

6 अधोलिखितवाक्येषु रे खांकिपदे षु सन्धिं वा सन्धिच्छे दं कुरुत- 1x4=4


(केवलं प्रश्नचतष्ु टयम ् )
(i) किन्तु बालस्य अत्र रक्षकः नास्ति l

(ii) हे श्वेतकेतो ! गरु


ु ं प्रति गच्छ अध्ययन + अर्थम ् ।

(iii) भवत ्+ राज्ये निवस्तम


ु ् इच्छामि ।

(iv) धन्या महीरुहा येभ्यो निराशा यान्ति नार्थिनः ।

(v) धनसेनयोः अभावे दे शरक्षणाय पर्याकुलाः स्मः ।

7 उचितविकल्पेन रिक्तस्थानानि परू यत- ( केवलं प्रश्नचतष्ु टयम ् ) 1x4=4


(i) यद् वस्तु भवदीयम ् अस्ति , तत ्______ अर्पय ।
(क)मया (ख) माम ्
(ग) मह्यम ् (घ)मत ्

(ii)______अभितः जीवाः सन्ति।


(क) चन्दनवक्ष
ृ ः (ख) चन्दनवक्ष
ृ म्
(ग) चन्दनवक्ष
ृ स्य (घ) चन्दनवक्ष
ृ ाय

(iii) तदा ____ उपरि एका बलाका विष्ठाम ् उदसज ृ त्।


(क) मनु ेः (ख) मनिु ः
(ग) मनि
ु ना (घ) मनिु भ्यः

(iv) हे जननि ! अलम ् _____।


(क) चिन्ता (ख) चिन्तया
(ग) चिन्तायै (घ) चिन्तायाः

(v) अहम ् _____ सह क्षिप्रम ् आगमिष्यामि ।


(क) पिता (ख) पित्रा
(ग) पितरम ् ( घ) पितःु

8 उचितं विकल्पं चित्वा रिक्तस्थानपर्ति


ू ः कुरुत-( केवलं प्रश्नचतष्ु टयम ् )1x4=4
(i) आज्ञा ________हि अविचारणीया ।
(क) गरु
ु भिः (ख) गरूु णाम ्
( ग) गरु ु न् (घ) गरु वः

(ii) ______ कालिदासः सर्वोत्तमः आसीत ् ।


(क) कविभ्यः (ख) कवयः
(ग) कवी (घ) कविषु

(iii) महर्षेः ______ पत्र


ु ः श्वेतकेतःु आसीत ् ।
(क) आरुणिः (ख) आरुणिम ्
(ग) आरुणेः (घ) आरुण्या

(iv)_______दृष्ट्वा पण्डिताः तारस्वरे ण अहसन ् ।


(क) अष्टावक्रः (ख)अष्टावक्रम ्
(ग) अष्टावक्रस्य (घ)अष्टावक्रेण
(v)______पतिव्रता स्त्री तम ् अवदत ् ।
(क) सः (ख) सा
(ग) तेन (घ) तानि

9 उचितविकल्पेन रिक्तस्थानानि परू यत- ( केवलं प्रश्नचतष्ु टयम ् )1x4=4

(i) संसारे विद्याहीनाःन ______।


(क)शोभते (ख)शोभेते
(ग) शोभे ( घ) शोभन्ते

(ii) भवान ् सत्यम ्_____ ।


(क) वदतु (ख) वदन्तु
(ग) वदानि (घ) वद

(iii) उज्ज्यिन्याम ् माधवो नाम विप्रः ______ ।


(क) आसीः (ख) आसीत ्
(ग) आस्ताम ् (घ) आसम ्

(iv) तदा अन्यः कश्चित ् श्राद्धार्थं वत्त


ृ ः ______।
(क) भवेताम ् (ख) भवेयःु
(ग) भवेः (घ) भवेत ्

(v) ततः ते श्रेयो भविष्यति त्वं च निरहं कारं _________।


(क) भविष्यति (ख)भविष्यसि
(ग) भविष्यामि (घ) भविष्यावः

10 अधोलिखितवाक्येषु रे खांकितपदानां प्रकृति-प्रत्ययौ संयोज्य विभज्य वा उचितम ्


उत्तरम ् विकल्पेभ्य चित्वा लिखत- ( केवलं प्रश्नत्रयम ् ) 1x3=3

(i) गम ्+शत ृ पिपीलको याति योजनानाम ् शतान्यपि ।


(क) गच्छन ् (ख) गच्छन्तौ
(ग) गच्छन्तम ् (घ) गच्छता

(ii) सा साध्वी भर्तुः शश्र


ु ष ू ां कृत्वा मन
ु ेः अन्तिकम ् आगता ।
(क) कृ+ क्त्वा (ख) कृ+कत्वा
कृ+तम
ु न
ु ् (घ) कृ+शत ृ

(iii) तस्य वक्रदे हं वि+ईक्ष् ल्यप ् पण्डिताः तारस्वरे ण अहसन ् ।


(क) वीक्षय (ख) वीक्ष्य
(ग) विक्ष्य+ल्यप ् (घ) निरीक्ष्य

(iv) न शक्नोमि स्वचक्षुषा दृश ्+तम


ु नु ् स्वदे शदर्दु शाम ् ।
(क) द्रष्टुम ् (ख) दर्शितम ु ्
(ग) दृश ्+क्त्वा (घ) प्रदर्श्य

11 उचिताव्ययपदै ः रिक्तस्थानानि परू यत-( केवलं प्रश्नचतष्ु टयम ् )1/2x4=2

(i) विलयं व्रजन्तु ______विपदः।

(ii) त्वया मया ______राजसभां गन्तव्यम ् ।

(iii) येन श्रत


ु न
े अन्यत ् अश्रत
ु म ् ____ श्रत
ु म ् भवति ।

(iv) हे मन
ु े!______ अहम ् बलाकेति ।

(v) एतत ् ____ इव ज्ञातम ् अनया ।

मञ्जष
ू ा- (सह, कथम ्, च, अपि, न )

12 उचितसंख्यावाचकपदै ः रिक्तस्थानानि परू यत- (केवलं प्रश्नचतष्ु टयम ् )1/2x4=2

(i) पथि
ृ व्यां_______ (3) रत्नानि , जलमन्नं सभ
ु ाषितम ्।

(ii) भामाशाहः ______(12) वर्षाणि यावत ् पञ्चविंशतिसहस्रसैनिकानां पालयित्रीं सम्पत्तिं


प्रतापाय अर्पयति ।

(iii) ______(1) चन्द्रः तमो हन्ति।

(iv) मम समीपे ______ (99) रुप्यकाणि सन्ति।


(v) तत्र वक्ष
ृ े _____( 45) निम्बवक्ष
ृ ाः सन्ति।
घ भागः
पठितावबोधनम ् 30अङ्काः
13.अधोलिखितं गद्यांशं पठित्वा प्रदत्तप्रश्नानाम ् उत्तराणि संस्कृतेन लिखत- 5

ततश्च सा साध्वी भर्तुः शश्रु ष ू ां कृत्वा मिक्षामादाय मन ु ेः अन्तिकम ् आगता । स च मनि ु ः


बद्धाञ्जलिः तामवदत ् - कथं त्वया अनन्यगोचरो बलाकावत्त ृ ान्तो ज्ञात इति ब्रहि
ू , ततो
भिक्षां ग्रहीष्ये ।" साऽवदत ् - "मन ु े! न भर्तृसेवायाः अपरं कञ्चन धर्मं करोम्यहम ्।
तत्प्रसादे न एव मे एतादृशं विज्ञानम ्। किञ्च इतः धर्मव्याधाख्यं कञ्चन मांसविक्रयिणं
गत्वा एतत ् पच् ृ छ । ततस्ते श्रेयो भविष्यति त्वं च निरहङ्कारः भविष्यसि इति।" एवं
सर्वविदा पतिव्रतया अभिहितः गह ृ ीतातिथिसत्कारः स मनि ु ः तां प्रणम्य तद्गहृ ाद्
निरगच्छत ् ।

( अ) एकपदे न उत्तरत-(केवलं प्रश्नद्वयम ् ) 1/2x2=1

(i) साध्वी किम ् आदाय मन ु ेः अन्तिकम ् आगता?


(ii) मांसविक्रयिणः किं नाम आसीत ् ?
(iii) सः काम ् प्रणम्य निरगच्छत ् ?

( आ) पर्ण
ू वाक्येन उत्तरत- ( केवलं प्रश्नद्वयम ् ) 1x2=2

(i) सा मनिु ं किम ् अवदत ् ?


(ii) कीदृशः मनिु ः तां प्रणम्य निरगच्छत ् ?
(iii) मन
ु ेः अन्तिकम ् का आगता ?

(इ) निर्देशानस
ु ारम ् उत्तरत- ( केवलं प्रश्नद्वयम ् ) 1x2=2

ु छे दे 'समीपम ्' इत्यस्य पदस्य कृते किं पदम ् आगतम ् ?


(i) अनच्

(ii) बद्धाञ्जलिः इति विशेषणपदस्य किं विशेष्यपदं किम ् ?

(iii) 'कल्याणम ्' इत्यस्य पदस्य अनच्


ु छे दे कः पर्यायः प्रयक्
ु तः ?

(iv) 'भविष्यति' इति क्रियापदस्य कर्तृपदं किम ् ?


14 अधोलिखितं पद्यांशं पठित्वा प्रदत्तप्रश्नानाम ् उत्तराणि संस्कृतेन लिखत- 5

धत्ते भरं कुसम


ु पत्रफलावलीनां घर्मव्यथां वहति शीतभवां रुजं च ।
यः सर्वमर्पयति चान्यसख ु स्य हे तोः तस्मै वदान्यगरु वे तरवे नमोऽस्तु ॥

(अ) एकपदे न उत्तरत-(केवलं प्रश्नद्वयम ् ) 1/2x2=1


(i) कस्मै नमः अस्तु ?
(ii) आवलीनां भारं को धत्ते ?
(iii) कः अन्यसख
ु स्य हे तःु भवति ?

( आ) पर्ण
ू वाक्येन उत्तरत-( केवलं प्रश्नद्वयम ् ) 1x2=2
(i) तरुः केषां भरं धत्ते ?
(ii) तरुः कस्य हे तोः सर्वम ् अर्पयति ?
(iii) अत्र आदरणीयः अस्ति ?

( इ) निर्देशानस
ु ारम ् उत्तरत -( केवलं प्रश्नद्वयम ्) 1x2=2

(i) श्लोके 'दःु खस्य' इति पदस्य कः विपर्ययः आगतः ?

(ii) वदान्यगरु वे तरवे नमोSस्तु ।अत्र क्रियापदं किम ् ?

(iii) वदान्यगरु वे इति विशेषणपदस्य विशेष्यपदं किम ् ?

15 अधोलिखितं नाट्यांशं पठित्वा प्रदत्तप्रश्नानाम ् उत्तराणि संस्कृतेन उत्तरत-5

भोजः - साधु कुविन्द ! साधु ! कीदृशी लालित्यपर्णा ू पदयोजना 'कवयामि वयामि यामि'
धन्योऽसि त्वम ् । अहमपि आत्मानं धन्यं मन्ये, यस्य राज्ये तन्तव ु ायाः, कुम्भकाराः,
अयस्काराः, शिल्पिनः, श्रमिकाः अपि ललितां काव्यरचनां कुर्वन्ति ।
(तन्तव ु ायं प्रति) भोः कविवर ! गह ृ ाण पष्ु कलं स्वर्णमयं पारितोषिकम ् । निजम ् आवासं च
प्रयाहि । (मन्त्रिणं प्रति) पण्डितः लक्ष्मीधरः मम राजभवने वासयितव्यः, यावद् अपरा
व्यवस्था न भवति ।
(राजा तन्तव ु ायाय स्वर्णमद्र ु ाः अर्पयति । तन्तव ु ायः पारितोषिकं प्राप्य प्रसन्नवदनः
राजानम ् अभिवाद्य गह ृ ं प्रयाति) ।

1. एकपदे न उत्तरत- (केवलं प्रश्नद्वयम ् ) 1/2x2=1


(i) कुविन्दे न कीदृशी पदयोजना कृता ?
(ii) तन्तनू ां वयनं के कुर्वन्ति ?
(iii) गह
ृ ाण पष्ु कलं स्वर्णमयं पारितोषिकम ् इति कः कथयति ?

II. पर्ण
ू वाक्येन उत्तरत- ( केवलं प्रश्नद्वयम ् ) 1x2=2

(i) राजा तन्तवु ायाय काः अर्पयति ?


(ii) राजा आत्मानं कीदृशं मन्यते ?
(iii) पण्डितः लक्ष्मीधरः कुत्र वासयितव्यः ?

III. निर्देशानस
ु ारम ् उत्तरत- (केवलं प्रश्नद्वयम ् ) 1x2=2

(i) 'शोभनाम ्' इत्यर्थे अत्र किं पदं प्रत्यक्


ु तम ् ?

(ii) 'गच्छ' इति क्रियापदस्य कः पर्यायः अस्मिन ् नाट्यांशे प्रयक्


ु तः ?

(iii) अत्र 'मन्ये' क्रियायाः कर्तृपदं किं वर्तते ?

16 रे खांकितपदानि आधत्ृ य प्रश्ननिर्माणं कुरुत-( केवलं प्रश्नपंचकम ् )1x5=5

(i) पथि
ृ व्यां त्रीणि रत्नानि जलमन्नं सभ
ु ाषितम ् ।

(ii) अत्रान्तरे ब्राह्मणोSपि श्राद्धं गह


ृ ीत्वा गह
ृ म ् उपावत्त
ृ ः।

(iii) एवम ् अणिम्नः स्थल


ू ं जगद् उद्भवति ।

(iv) राजनियमः तु मया पालनीयः ।

(v) वक्ष
ृ ेभ्यः अर्थिनः निराशा न यान्ति ।

(vi) परु ा महातपा नाम वनवासी मनि


ु ः आसीत ् ।

17 मञ्जष
ू ातः समचि
ु तपदानि चित्वा अधोलिखित-श्लोकस्य अन्वयं परू यत- 1/2x4=2
सद्भिस्तु लीलया प्रोक्तं शिलालिखितमक्षरम ् ।
असद्भिः शपथेनापि जले लिखितमक्षरम ् ।।

अन्वयः- (i)_____ तु लीलया प्रोक्तम ् (ii)______अक्षरम ् ( इव भवति ) । असद्भिः


शपथेन (iii)______ जले (iv)______ अक्षरम ् ( इव भवति )

मञ्जष
ू ा-
( अपि , लिखितम ् , शिलालिखितम ् , सद्भिः )

अथवा
मञ्जष
ू ासहायतया श्लोकस्य भावार्थे रिक्तस्थानानि परू यित्वा पन
ु ः लिखत-
1/2x4=2

सहसा विदधीत न क्रियामविवेकः परमापदां पदम ् ।


वण
ृ ते हि विमश्ृ यकारिणं गण
ु लब्ु धाः स्वयमेव सम्पदः ।।

भावः- सहसा कोSपि (i)_____ न कुर्यात ् । अविवेकः तु परम ् (ii)____ पदम ् भवति ।
गण
ु लब्ु धाः जनाः (iii)______ स्वयमेव हि (iv)______ वण
ृ ते ।

मञ्जष
ू ा- ( विमश्ृ यकारिणम ् , सम्पदः, आपदां , क्रियाम ् , )

18 अधोलिखितवाक्येषु रे खांकितपदानां प्रसंगानक


ु ु लम ् उचितार्थं चित्वा लिखत- ( केवलं
प्रश्नचतष्ु टयम ् ) 1x4=4

(i) फलोद्गमैः तरवः नम्राः भवन्ति ।


(क) महीरुहाः (ख) वक्ष
ृ ाः
(ग) शाखाः (घ) गरु वः

(ii) स्वधर्मं चर , येन आशु परं श्रेयः अवाप्स्यसि ।


(क) कठिनतया (ख) शनैः
(ग) शीघ्रम ् (घ) सरलतया

(iii) महाराणाप्रतापः अरण्ये उपविष्टः अस्ति।


(क) गह ृ े (ख) सिंहासने
(ग) हस्तिः (घ) वने
(iv) सः जातः येन जातेन वंशः समन्
ु नतिं याति ।
(क) रुग्णेन (ख) मत्ृ यनु ा
(ग) जन्मना (घ) स्वाथ्येन

(v) सः तु दरिद्रः भवति यस्य तष्ृ णा विशाला भवति ।


(क) अनिच्छा (ख) इच्छा
(ग) वाक् (घ) ज्योतिः

19 मञ्जष
ू ायाः सहायतया कथापर्तिं
ू कृत्वा कथां पन
ु ः लिखत- 1/2x8=4

अष्टावक्रः(i)______ अपच् ृ छत ्- मातः ! क्व आस्ते मे पिता । अष्टावक्रस्य पिता राजर्षेः


जनकस्य सभां विद्वद्भिः सह (ii)______गतः आसीत ् । मात्रा अनम ु तः अष्टावक्रः
राजसभां गतः । (iii)_____तं वक्रदे हं दृष्ट्वा हसितम ु ् प्रारभन्त । अष्टावक्रः पण्डितानां
चेष्टितं दृष्ट्वा उच्चैः अहसत ् । (iv)_______अपच् ृ छत ् - भो अष्टावक्र ! किमर्थं
हससि? (v)______उवाच- एते मे वक्रदे हं दृष्ट्वा हसन्ति , अहम ् एतेषां मर्ख ू तां दृष्ट्वा
हसामि । हे राजन ् ! किं वक्रे (vi)______आत्मा अपि वक्रः भवति। अष्टावक्रस्य
शास्त्रपाण्डित्येन सर्वे प्रभाविताः अभवन ्। जनकः प्रभावितः (vii)______अष्टावक्राय
किमपि दातम ु ् इच्छति । जनकः पराजितः भत्ू वा अष्टावक्रस्य पादयोः (viii)_______।
मञ्जष ू ा-
( शास्त्रार्थाय, भत्ू वा , पण्डिताः ,अष्टावक्रः , जनकः , दे हे , पतति , मातरम ् )
संस्कृतम ्
आदर्शप्रश्नपत्रम ्
कक्षा नवमी
( 2023 - 2024 )
अवधिः - होरात्रयम ् पर्णां
ू ङ्काः 80
अंकविभाजनम ् उत्तरसंकेताश्च

क्रमसं उत्तरसंकेताः अंकविभाजन योगः


ख्या म्
1 खण्डः क ( पठितावबोधनम ् ) 10 अङ्काः

(अ) एकपदे न उत्तरत-(केवलं प्रश्नद्वयम ् ) 1x2=2 2


(i)सतां
(ii) सामाजिकः
(iii)सत्संगतिः
(आ) पर्णू वाक्येन उत्तरत- (केवलं प्रश्नद्वयम ् ) 2x2=4 4
(i) सत्संगकारणात ् दर्ज
ु नाः अपि सज्जनाः भवन्ति।
(ii)सामाजिकः प्राणी अपि मानवसंसर्गेण विना जीवितमु ् न शक्नोति।
(iii) सर्वे एव संगतिम ् इच्छन्ति।

(इ) मानवः संसर्गशीलः / सामाजिकः प्राणी ,अनच्


ु छे दाधारितम ् 1x1=1 1
अन्यत ् किमपि शीर्षकं भवितंु शक्नोति।

(ई) निर्देशानसु ारम ् उत्तरत-( केवलं प्रश्नद्वयम ् ) 1x3=3 3


(i)(क) सतां
(ii)(क) मानवः
(iii)(घ) सदा
(iv) (घ) प्राणी

2 खण्डः ख 5
रचनात्मककार्यम ् 15 अङ्काः 1/2x10=
पत्रलेखनम ् - 5
(i) दिल्लीनगरतः (vi) पश्चातापं
(ii) शभु ाशीषः (vii)समयस्य
(iii) अस्तु (viii) निरर्थकः
(iv) महत्त्वपर्णं
ू (ix) प्रतीक्षायां
(v) भवन्ति (x) अग्रजः
3 चित्रानरू
ु पम ् छात्राः स्वेच्छया पंचवाक्यानि लेखिष्यन्ति। 1x5=5 5
अथवा

विषयानरू
ु पम ् छात्राः स्वेच्छया पंचवाक्यैः सह अनच्
ु छे दं लेखिष्यन्ति।

कथापर्ति
ू ः-
1/2x10=5 5
(i)मर्यादापरु ु षोत्तमः (vi)वनम ्
(ii) राज्ञः (vii)रावणः
4 (iii) पितःु (viii) रावणं
(iv) निवासम ् (ix)लंकाराज्यम ्
(v) पत्नी (x) जीवनकथा

अथवा
संवादः- 1x5=5 5

(i) दीपावली हिन्दन ू ां प्रमख


ु ः उत्सवः अस्ति।
(ii) अस्मिन ् दिने श्री रामः अयोध्याम ् आगच्छत ् ।
(iii)अस्मिन ् दिने जनाः स्वगह ृ ाणि भष
ू यन्ति । दीपकान ्
प्रज्ज्वालयन्ति।
(iv)दीपावल्याम ् रात्रौ किं भवति ?

(v)लक्ष्मीः धनस्य दे वी अस्ति।

5 खण्डः ग अनप्र
ु यक्
ु तव्याकरणम ् अङ्काः 25

उच्चारणस्थानम ्- 1/2x4=2 2
(i) आ
(ii) नासिक्यः
(iii) ओष्ठौ
(iv) दन्त्यः
(v) फ्

6 सन्धिः- (केवलं प्रश्चतष्ु टयम ् ) 1x4=4 4


(i) न + अस्ति
(ii) अध्ययनार्थं
(iii) भवद्राज्ये
(iv)येभ्यः+ निराशा
(v) परि +आकुलाः
1x4=4 4
7 कारक-उपपदविभक्तिः- (केवलं प्रश्चतष्ु टयम ्)
(i) (ग)मह्यम ्
(ii) (ख) चन्दनवक्ष
ृ म्
(iii)(क) मन
ु ेः
(iv)(ख) चिन्तया
(v)(ख) पित्रा

1x4=4 4
8 शब्दरूपाणि- (केवलं प्रश्नचतष्ु टयम ् )

(i) (ख) गरू


ु णाम ्
(ii)(घ) कविषु
(iii) (ग) आरुणेः
(iv)(ख) अष्टावक्रम ्
(v) (ख) सा

9 धातरू
ु पाणि- (केवलं प्रश्नचतष्ु टयम ् ) 1x4=4 4
(i) (घ) शोभन्ते
(ii) (क) वदतु
(iii)(ख) आसीत ्
(iv)(घ) भवेत ्
(v)(ख) भविष्यसि

10 प्रत्ययः-(केवलं प्रश्नत्रयम ् ) 1x3=3 3

(i) (क) गच्छन ्


(ii)(क) कृत्वा
(iii)(ख) वीक्ष्य
(iv)(क) द्रष्टुम ्
11 अव्ययपदानि -(केवलं प्रश्नचतष्ु टयम ् ) 1/2x4=2 2
(i) च
(ii) सह
(iii) अपि
(iv) न
(v) कथम ्

12 संख्या-(केवलं पश्नचतष्ु टयम ् ) 1/2x4=2 2


(i) त्रीणि
(ii) द्वादश
(iii) एकः
(iv) नवनवतिः
(v) पंचचत्वारिंशत ्

खण्डः घ पठितावबोधनम ् 30 अङ्काः 1/2x2=1 1


13 (अ) एकपदे न उत्तरत-( केवलं प्रश्नद्वयम ् )

(i) भिक्षाम ्
(ii) धर्मव्याधः
(iii) पतिव्रतां

(आ) पर्ण
ू वाक्येन उत्तरत-( केवलं प्रश्नद्वयम ् ) 1x2=2 2

(i) मनु ेः ! न भर्तृसेवायाः अपरं कञ्चन धर्मं करोम्यहम ्। तत्प्रसादे न एव


मे एतादृशं विज्ञानम ् ।
(ii) गहृ ीतातिथिसत्कारः स मनि ु ः तां प्रणम्य तद्गह
ृ ाद् निरगच्छत ् ।
(iii) मनु ेः अन्तिकम ् साध्वी आगता ।

(इ) निर्देशानस
ु ारम ् उत्तरत-(केवलं प्रश्नद्वयम ् ) 1x2=2 2

(i) अन्तिकम ्
(ii) मनि ु ः
(iii) श्रेयः
(iv) श्रेयः

14 (अ) एकपदे न उत्तरत- (केवलं प्रश्नद्वयम ् ) 1/2x2=1 1


(i)तरवे
(ii) वक्ष
ृ ः/तरुः
(iii)वक्ष
ृ ः /तरुः

(आ) पर्ण
ू वाक्येन उत्तरत-( केवलं प्रश्नद्वयम ् ) 1x2=2 2

(i) वक्ष
ृ ः कुसम ु पत्रफलावलीनां भरं धत्ते।
(ii) तरुः अन्यसख ु स्य हे तोः सर्वम ् अर्पयति ।
(iii) अत्र वक्ष
ृ : /तरुः आदरणीयः अस्ति।
1x2=2 2
(इ) निर्देशानस
ु ारम ् उत्तरत-(केवलं प्रश्नद्वयम ् )

(i) सख ु स्य
(ii) अस्तु
(iii) तरवे

15 (अ)एकपदे न उत्तरत-(केवलं प्रश्नद्वयम ् ) 1/2x2=1 1

(i) लालित्यपर्णा

(ii) तन्तव
ु ायाः
(iii) राजाभोजः
1x2=2 2
(आ) पर्ण
ू वाक्येन उत्तरत-(केवलं प्रश्नद्वयम ् )

(i) राजा तन्तवु ायाय स्वर्णमद्र


ु ाः अर्पयति ।
(ii) राजा आत्मानं धन्यं मन्यते ।
(iii) पण्डितः लक्ष्मीधरः मम राजभवने वासयितव्यः ।

(इ) निर्देशानस
ु ारम ् उत्तरत-(केवलं प्रश्नद्वयम ् ) 1x2=2 2

(i) ललिताम ्
(ii) प्रयाहि
(iii) अहम ्

16 प्रश्ननिर्माणम ्-(केवलं प्रश्नपंचकम ् ) 1x5=5 5

(i) कस्याम ्/कुत्र


(ii) कुत्र

(iii) कस्मात ्

(iv) कः

(v) के

(vi) किम ्

17 अन्वयः- 1/2x4=2 2
(i) सद्भिः
(ii) शिलालिखितम ्
(iii) अपि
(iv) लिखितम ्

अथवा
भावार्थः-
(i) क्रियाम ्
(ii) आपदां
(iii) विमश्ृ यकारिणम ्
(iv) सम्पदः

18 प्रसंगानक ु ु लम ्- 1x4=4 4
(i) (क) महीरुहाः
(ii) (ग) शीघ्रम ्
(iii) (घ) वने
(iv)(ग) जन्मना
(v)(ख) इच्छा

19 कथापर्ति
ू ः- 1/2x8=4 4

(i) मातरम ्
(ii) शास्त्रार्थाय

(iii) पण्डिताः

(iv) जनकः

(v) अष्टावक्रः

(vi) दे हे

(vii) भत्ू वा

(viii) पतति

You might also like